introduction - university of victoria -...

124
INTRODUCTION Definition Castelles: The branch of law of each province or territory (including federal law) which in a case involving at least one legally relevant foreign element connecting it with more than one legal unit determines before the courts of which unit the suit should be brought and by the law of which unit each issue should be decided. The branch of law of each province or territory – can vary from province to province, will definitely vary between countries; there is NO global law or international understanding. including federal law – NOTE: while you can have conflicts of law issue between BC and Alberta, cannot have one between BC and Feds (because then is a constitutional issue) which in a case – this course deals with civil litigation, criminal litigation is dealt with under statute – conflict resolved by statute or public international law involving at least one legally relevant foreign element connecting it with more than one legal unit – has to be some foreign element! determines before the courts of which unit the suit should be brought – determines whether the province’s civil procedure rules apply and by the law of which unit each issue should be decided – which law should apply Structure Conflicts is a set of rules (or series of argumentative propositions) that overarch or parallel our regular domestic law system – procedural rather than substantive law Which forum has jurisdiction over a particular dispute and should hear the matter? Two parts: Does the court have the authority to hear the suit? Is it the most appropriate forum? Whose law applies? THIS IS WHERE CONFLICT ARISES (where there is a difference or inconsistency in the laws of the forum and foreign states). Where can the judgement be enforced? Two parts:

Upload: trankien

Post on 22-Mar-2018

223 views

Category:

Documents


8 download

TRANSCRIPT

Page 1: INTRODUCTION - University of Victoria - Web.UVic.caweb.uvic.ca/~lssweb/wp-content/outline_uploads/76... · Web viewDow Corning (2000) (BCCA) Facts: Class actions for negligence in

INTRODUCTIONDefinitionCastelles: The branch of law of each province or territory (including federal law) which in a case involving at least one legally relevant foreign element connecting it with more than one legal unit determines before the courts of which unit the suit should be brought and by the law of which unit each issue should be decided.

The branch of law of each province or territory – can vary from province to province, will definitely vary between countries; there is NO global law or international understanding. including federal law – NOTE: while you can have conflicts of law issue between BC and Alberta, cannot have one between BC and Feds (because then is a constitutional issue) which in a case – this course deals with civil litigation, criminal litigation is dealt with under statute – conflict resolved by statute or public international law involving at least one legally relevant foreign element connecting it with more than one legal unit – has to be some foreign element!determines before the courts of which unit the suit should be brought – determines whether the province’s civil procedure rules apply and by the law of which unit each issue should be decided – which law should apply

StructureConflicts is a set of rules (or series of argumentative propositions) that overarch or parallel our regular domestic law system – procedural rather than substantive law

Which forum has jurisdiction over a particular dispute and should hear the matter? Two parts:

Does the court have the authority to hear the suit? Is it the most appropriate forum?

Whose law applies? THIS IS WHERE CONFLICT ARISES (where there is a difference or inconsistency in the laws of the forum and foreign states). Where can the judgement be enforced? Two parts:

Litigation strategy – no sense getting a judgement if you can’t enforce it where the money is.

Overview of ConflictsCause of ActionSuit – court jurisdiction

Need to have jurisdiction over both the person and the subject matterNeed for service ex juris

Jurisdictional Challenge – court’s discretion to exercise jurisdiction

Page 2: INTRODUCTION - University of Victoria - Web.UVic.caweb.uvic.ca/~lssweb/wp-content/outline_uploads/76... · Web viewDow Corning (2000) (BCCA) Facts: Class actions for negligence in

Forum non conveniens – may not be a convenient jurisdiction to hear the case, therefore, would be inappropriate for the court to exercise its jurisdiction on a particular caseAnti-suit - person asks a court to in the plaintiff’s home jurisdiction to enter an injunction restraining the plaintiff from continuing or initiating the lawsuit in the foreign jurisdiction

Choice of LawJurisdictional categories and connecting factors:

Tort: lex loci delecti (where the tort took place)Contract: proper law of contract (interpretive)

Judgement – recognition and enforcement of the judgement

Approaches to Conflicts of Laws Public international law deals with international rules that are designed to deal with states whereas conflicts rules are domestic court rules that regulate private individuals in cases with an international element

Principle of TerritorialityGeneral rule: every nation possess as exclusive sovereignty and jurisdiction within its own territory

Effect: rules of the state are binding on all property, people, and contracts made within its territory (specific geographic location)Poblem (Canada): may be overlapping between federal, provincial, and First Nations

BUT no state can directly affect or bind property beyond its territory, or non-residents (can be supreme within BC, but problem if try to move outside of BC)How to reconcile territoriality and the application of foreign law: (Ted says none really work)

Comity – do justice in order that justice may be done to us in returnVested rights (American, rejected in Canada but sometimes raised heartstrings argument) Local law – foreign law isn’t really being applied because it becomes the law of the forumGovernment interest analysis – problem: requires the court to look at the policy in the legal units (evaluating competing government interests) – courts are always going to understand the local policy better than the foreign and so likely to win.

Whatever force and obligations the laws of one country have in another depends solely upon the laws and municipal regulations of the latter (need express or tacit consent)

ComityGeneral rule: recognition of foreign law depends on comity the law of no country can have effect as law beyond it’s own territory unless by permission of another state Application of foreign law is not a matter of caprice or option (i.e. will not apply foreign law merely out of consideration for another state).

Page 3: INTRODUCTION - University of Victoria - Web.UVic.caweb.uvic.ca/~lssweb/wp-content/outline_uploads/76... · Web viewDow Corning (2000) (BCCA) Facts: Class actions for negligence in

Flows out of the impossibility of otherwise determining whole classes of cases without gross inconvenience and injustice to litigants

Comity: deference to foreign laws seen as an attempt to promote international harmony by accommodating the views of a foreign sovereign in the expectation of receiving reciprocal treatment - principle of enlightened self-interest

Vested Rights All rights must be created by some law (are political rather than social creations) A right having been created by the appropriate law, the recognition of its existence should follow everywhere (where the right arose, this should follow the individual around) There is no exception from territorial law, instead local law simply recognizes that a right has become vested in an individual under the foreign law at the time when the individual was subject to the foreign law. Emphasized the individual’s entitlement to his vested right rather than the courts’ politeness or concession to foreign sovereigns

Local Law TheoryThe forum incorporates foreign law, and is then able to apply it as domestic law Allows the conclusion that the court only enforces rights created by its own law – what you are really doing isn’t applying foreign law, just making domestic law like foreign law

Governmental Interest Analysis Argues that we would be better off without choice of law rulesSuggest a method to follow: normally apply domestic law (even where there is a foreign element), consider the policy of the domestic and foreign laws, apply the foreign law where the forum state has no interest in the application of its policy but the foreign state does. Where both states have an interest (or the foreign state has no interest), should apply the law of the forum state. In line with modern theories that see conflict of laws as having to do with the familiar CL task of deciding the appropriate scope of any rule given its underlying rationale or purpose

Courts consider whether the local law, which has been developed to respond to local situations, ought to be modified in light of a foreign element Courts should consider the underlying policy or purpose the law is meant to serve and then ask whether the law should be applied

Conflicts and the ConstitutionSovereignty and Extra-territoriality

The ability to legislate extra-territorially is a matter of sovereignty. Statute of Westminster, 1931: Gave Canada the ability to legislate with extra-territorial effect; previously, had lacked the ability as a colony of the British Empire.Provinces certainly do not have the power to legislate with extra-territorial effect

Page 4: INTRODUCTION - University of Victoria - Web.UVic.caweb.uvic.ca/~lssweb/wp-content/outline_uploads/76... · Web viewDow Corning (2000) (BCCA) Facts: Class actions for negligence in

Churchill Falls (Labrador) Corp. Ltd. v. AG of Newfoundland (1984)Significance: The leading constitutional case dealing with extraterritoriality - looks at ability of provincial government to legislate on matters that may be within the province but might also have consequence external of the province.

Facts: Contractual arrangement in relation to the Churchill Falls river, development of hydroelectric facility by Hydro Quebec through subsidiaries, to produce cheaper electricity for population. This contract stated that the laws of Quebec will apply. Involved two parts: fixed price agreement and service term. Quebec paid above market price for the first few years, but eventually because of the fixed price they were paying below market price. NFL was unhappy and attempted to get out of the contract. Quebec found it didn’t need all the electrical supply units and decided to sell excess off to US at higher price.

Hydro Quebec argued that: Argued that while all that would be taken under the Act is physically situated within the province of NFL, the effect would be to destroy lawfully acquired civil rights outside the province – any provincial legislation that has extraterritorial effect is ultra viresArgued the Reversion Act was aimed at the destruction of the rights of HQ under the power K, rights that were situated outside of NFL. The true purpose and intent of the legislation, its “pith and substance”, governs the issue of territorial limitation just as it does in other constitutional cases involving division of powers.The Act is beyond the legislative competence of the province of NFL

Court looked at precedents:Disagreement in case authorities as to the test which should be applied in determining the constitutional validity of a provincial statue of extra-territorial effect. Two lines of cases:

Extra-provincial legislation was ultra viresProvincial legislation could incidentally affect matters outside of the province

Royal Bank case (1913)Held that any provincial enactment which is not wholly confined in its effect to that province would be therefore ultra viresNo differentiation between statutes which are directed at extra-provincial rights and statutes which only incidentally affect those rights.

Ladore et al v. Bennett et al (1939)Lord Atkin held the pith and substance of the Acts was in relation to municipal institutions within the province and were justified as having been passed in relation to local works and undertakings under British N .America Act 1867

“…although they affect rights outside the Province they only so affect them collaterally, as a necessary incident to their lawful POGG with the Province.”

Day v Victoria [1938] applied the pith and substance test to territoriality – whether it is in the derogation from, or elimination of extra-provincial rights.

Even if it is cloaked with proper const form, it will be ultra vires.

Page 5: INTRODUCTION - University of Victoria - Web.UVic.caweb.uvic.ca/~lssweb/wp-content/outline_uploads/76... · Web viewDow Corning (2000) (BCCA) Facts: Class actions for negligence in

Rule: Where the P&S of the provincial enactment is in relation to matters which fall within the field of provincial legislative competence, incidental or consequential effects on extra-provincial rights will not render the enactment ultra vires

BUT beware of colourability: Where the P&S of the enactment is the derogation from or elimination of extra-provincial rights the, even if cloaked in proper constitutional form, it will be ultra vires

Held: Supreme Court undertook a pith and substance analysisThe Reversion Act is a colourable attempt to interfere with the power K and thus to derogate from the rights of Hydro Quebec to receive an agreed amount of power at an agreed priceWhile the assets being nationalized were in NFL, found that the primary purpose was to undermine the contract.Interfered with the contractual rights within Quebec and was thus beyond the constitutional reach of NFL.It was more than an incidental or consequential impact (prima facie ultra vires).

Williams v. Canada (1992)Significance: When the court looked at where something occurred, or where rights were situated, they looked at it in a constitutional context, but used conflict of laws rules to provide the answer:

Facts: Related to the immunity from taxation and execution of judgments against an aboriginal person on an Indian Reserve under and Indian Act.

Issue: Whether the entitlement to an unemployment insurance cheque is a right on the reserve, or is it a right that relates to something that is situated off the Reserve?

Conflicts rule: Entitlement to cheque equivalent to entitlement to salary or debt Location of the debt/property is at the residence of the debtor (person paying).

Problem: The cheque came from the Federal Crown (Ministry of Employment), so if apply conflicts rules analogy (of debtor paying salary) in interpreting the Indian Act, location is not on the Reserve no immunity under s. 8 of Indian Act

Question for court was whether they were to apply the conflicts rule to this case:

Cautionary word to tell us that the court today may not always be willing to take advantage of set COL rules, when in a different context. Is reasonable for the general purposes of conflicts of laws, however had to enquire as to the utility for the purposes underlying the exemption from taxation in the Indian ActCt also said were dealing with a different context which will nec bring about diff policy considerations that don’t come up with debtors or even employers. Should not be bound by this.If use too widely, may have consequence that have to use this in a set way (can’t change the conflict rule for fear of upsetting other areas of law) – conflict law will be “locked in”

Page 6: INTRODUCTION - University of Victoria - Web.UVic.caweb.uvic.ca/~lssweb/wp-content/outline_uploads/76... · Web viewDow Corning (2000) (BCCA) Facts: Class actions for negligence in

Sovereignty and the Federal StateIn the past 20 years, the Supreme Court has essentially said “let’s get rid of the Brits” as a concept. Canada is a federal state, which is very different from the imperialistic, unitary British perspective.

Starting in the 1990s, the Canadian approach to conflict of laws have evolved significantly. The SCC has held that to be valid, statutory or judicial conflicts of law must conform to the demands of territoriality (traditional) and the principles of order and fairness (new).Has been influenced by two American principles:

Due process: 14th Amerndment guarantees Americans the right against loss of life, liberty or property without due process of law. Limits what the state courts can do to exert power over a non-resident defendant. Full faith and credit (more important): if a court in one state issues a court order that must be enforced in another state, then full faith and credit means that the second state must respect the courts of the other state.

In Canada, there is no direct phraseology of full faith and credit. All we have is Charter s. 7.

Impact of FederalismConflicts rules must recognize that Canada is a federal state. Sister provinces must give effect to one another.BC courts are now obliged to pay greater respect to foreign judgments from other Canadian provinces, as long as there was a real and substantial connection. In Morguard, La Forest makes much of the idea of order and fairness – what should govern whether a court has jurisdiction is due (fair) process, and more importantly, full faith and credit.

Morguard Investments Ltd v. De Savoye (1990)Significance: SCC employed federalism principles to create new rule for recognition and enforcement of judgments interprovincially. The rule was expressly stated to be a common law rule as the case was not argued in constitutional terms, but the nature of the discussion was such that it provoked speculation as to its constitutional status.

Facts: Concerns the recognition to be given by courts in one province to a judgment of courts in another province, in a personal action brought to the latter province at a time when the D did not live there.

Morguard Investments was mortgagee of lands in Alberta. Appellant who was a resident of Alberta was the mortgagor, and subsequently moved to BC. Mortgages fell into default and Morguard brought actions in Alberta. Appellant took no steps to appear or defend the action. Mortgaged properties were sold but weren’t enough to cover the full amount.

Issue: Could judgment of court in Alberta be enforced against the appellant in BC?

Court’s Analysis (La Forest J):

Page 7: INTRODUCTION - University of Victoria - Web.UVic.caweb.uvic.ca/~lssweb/wp-content/outline_uploads/76... · Web viewDow Corning (2000) (BCCA) Facts: Class actions for negligence in

Examines history of conflicts in Canada noting common law strong territorial approach to recognition and enforcement of foreign judgments. In the traditional English perspective:

Sovereign states have exclusive jurisdiction in their own territoryState’s law has no binding effect outside its jurisdiction

Develops theories that depart from the territorial approach:Modern states cannot live in splendid isolation and do give effect to judgments given in other countries. Comity: Idea of “politeness” and “respect” of one state’s judgment process.

Says this is an old way of viewing comity. Rather, what has to be seen is “convenience, self-interest, necessity” – a common interest impels states to cooperate.

A person should not be allowed to avoid legal obligations arising in one province simply by moving to another province.

“The rules of private international law are grounded in the need to facilitate the flow of wealth, skills and people across state lines in a fair and orderly manner, in the recognition that the world is made of different global borders.”

Suggests traditional English approach was an exaggerated concern about perceived poor quality of justice served outside of BritainTalks about new world order and looks at the EU and US who both have more generous rules.

Interprovincial matters:Obvious intention of the Constitution is to create a single country – one of the central features of the Const Act 1867 was the creation of a common market.

S.121 of the Constitution – goal of the constitution is integration of a an economic nation

Promotion of mobility, common citizenship, social and economic stability.Need for cooperation between provincial courts in terms of enforcement of judgments

“The rules of comity as they apply between the provinces must be shaped to conform to the federal structure of the Const.”

Although other countries like the US and Australia use an explicit “full faith and credit clause”, in Canada this principle is now taken to be inherent (after Morguard). Various const and sub-const arrangements and practices make explicit clause unnecessary:

Judges are federally appointed and federally paid. Therefore is little risk that one judge would be more qualified than another (quality and consistency of judgments)SCC as final court controls all provinces within their respective unitsCanadian lawyers all adhere to the same code of ethicsProliferation of interprovincial law firms

Page 8: INTRODUCTION - University of Victoria - Web.UVic.caweb.uvic.ca/~lssweb/wp-content/outline_uploads/76... · Web viewDow Corning (2000) (BCCA) Facts: Class actions for negligence in

Enforcement of Judgments: Real and Substantial Connection Test

La Forest says there some limitations and qualifies “full faith and credit”Has to be weighed against fairness to the defendantCourts in one province should give full faith and credit to judgments given by a Court in another province so long as that Ct has properly or appropriately exercised jurisdiction in the action.“Meet demands of order and fairness to recognize a judgment given in a jurisdiction that had the greatest or at least significant contacts with the subject-matter of the action”Proceedings may have been brought where has no relation to the situation at hand and here a qualification has to be made

TEST:“Whether there is a real and substantial connection between the petitioner and the country/territory exercising jurisdiction”

The jurisdiction must have some significant context or relevance to the dispute, and if it doesn’t then the jurisdiction hasn’t been properly taken therefore enforcement doesn’t need to be made

Applying the RSC test to this case:Reasonable place for the action for the deficiencies to take place in Alberta. Land in Alberta = very substantial connection, lived in Alberta and was in business in AlbertaRSC connection between the damages suffered and the jurisdictionProper that Alberta Court had jurisdiction enforcement of judgment in BC is required

Hunt v T&N Plc (1993)Facts: Plaintiff in BC, defendant is a Quebec based company. BC decided that they had jurisdiction simplicitor and that they were the correct forum to here the matter. BC court orders the Quebec company to open itself up to discovery, in particular, to produce documents (located in Quebec) up to BC. However, the Business Concerns Records Act in Quebec prohibits the removal of documents from Quebec under a court order from outside of Quebec.

BC calls this blocking legislation – the entire purpose of the legislation is to block interference from external courts. The blocking statute would prohibit the removal from Quebec of documents of business concerns in Quebec have a lawful excuse for not presenting documents.

Issue: Whether the provisions of a Quebec Act, a “blocking statute”, provides a “lawful excuse” for Quebec defendants not to comply with a demand for discovery of documents (as required by the BC Rules of Court).

Effect: Can’t sue outside of Quebec unless you proceed without the right of discovery (which is impossible!). Quebec statute is unconstitutional in the way it impedes litigation. Does not respect principles of order and fairness as required by Morguard!

Page 9: INTRODUCTION - University of Victoria - Web.UVic.caweb.uvic.ca/~lssweb/wp-content/outline_uploads/76... · Web viewDow Corning (2000) (BCCA) Facts: Class actions for negligence in

Court Applies Morguard:Morguard stated need for a greater degree of recognition and enforcement of judgments given in other provinces. But a court must have reasonable grounds for assuming jurisdiction - there must be a “RSC” to the forum that assumed jurisdiction and gave judgment.

The discretion not to exercise jurisdiction must ultimately be guided by the requirements of order and fairness, not a mechanical counting of contacts or connections

Comity is to prevail and no longer is each province to live in isolation. When a company situated in one province willing engages in trade and commerce in another province, they must respect the court systems in other provinces

Should give “full faith and credit” to the judgments, including court orders, of the courts of sister provinces. It is beyond the power of the provincial legislation to override the structure of the federal country – must respect the constitutional standards of order and fairness.

Fed Parliament has power to legislate respecting the recognition and enforcement of judgments. This is related to the powers contained in the POGG clause.Subject to these overriding powers, provinces can legislate, subject to:

Principles in Morguard and The demands of territoriality as expounded in Churchill Falls

Held: Quebec Act was “constitutionally inapplicable” to other provinces. The law offends the basic structure of the Canadian federation and the principles of order and fairness.

Whole purpose of the blocking legislation was to impede litigation to ensure that foreign court orders would not be complied withQuebec legislation is mandatory – removes any sense of discretion on the part of the client (company) or on the part of the Quebec court. No exceptions allowed.

This is not about fairness, but about protection of the court. Although there may be situations where it would be reasonable to withhold business document, this cannot be the only option.

Effect of the law is to impede the substantive rights of other Canadians. Inconsistent with Canadian values; unfair to the BC litigants.

Result: Legislation is not struck down, but was found to be inapplicable to the other provinces. Legislation is still on the books as it applies to true foreigners (e.g. Americans).

Defendant ordered to comply with the BC court order. Quebec legislation found to be unconstitutional to the extent that it does not comply with the BC court order.

Tolofson v Jensen (1994)Significance: Constitutional limits not only exist on provincial legislation, but also on the provincial courts. This is particularly an issue with regard to choice of law.

Choice of law case: lex loci delicti

Page 10: INTRODUCTION - University of Victoria - Web.UVic.caweb.uvic.ca/~lssweb/wp-content/outline_uploads/76... · Web viewDow Corning (2000) (BCCA) Facts: Class actions for negligence in

Facts: Plaintiff (passenger) and driver were BC residents, travelling in a BC registered and insured vehicle. Injured in an accident in Saskatchewan.

General Rule: the law to be applied is that of the place where the tort occurred – lex loci delicti (law of the place of the wrong)

Flows logically from the general principle that a state has exclusive jurisdiction within its own territory, which other states must respect as part of international comity Advantages of this rule: certainty, ease of application, predictability, meets normal expectations (people expect activities to be governed by the place where they are when engaging in the activities)Equivocal: La Forest states the approach suggested also has the advantage of unquestionable conformity with the Constitution. Hasn’t really been demanded or decided either way concretely and is left for further comment.

Held: referred back to be decided on Saskatchewan law, as that was where the accident occurred.

Court JurisdictionThree questions that arise regarding court jurisdiction:

Does the court have jurisdiction at all? Jurisdiction simpliciter; BC legislation sometimes refers to it as territorial competence. Can the plaintiff use the court system?Does the court have jurisdiction over the defendant? – bulk of inquiry considers this question

Court has jurisdiction, but does it make sense for the court to exercise its discretion? Even if the court has jurisdiction, doesn’t necessarily mean that the court will exercise this jurisdiction.

Forum non conveniensAnti-suit injunctions

Jurisdiction in rem vs. in personamJurisdiction in personam must be de distinguished from jurisdiction in rem:

In rem: applies to immovable property, namely land. Court jurisdiction over immovable rests with the jurisdiction where the property lies.

E.g. if the land is in BC, then the BC courts have jurisdiction simpliciter. Also applies in the shipping world – admiralty law. Wherever the vessel is in port, that jurisdiction has jurisdiction over the vessel. In BC, under s. 5 BC Court Jurisdiction and Proceedings Something Act.

If the vessel is served or arrested in BC In personam: over a person for the purposes of claiming damages – mostly civil suits.

Does the forum where you are bringing the action have jurisdiction – can the plaintiff bring action?

Almost anyone who is in BC can sue here if they feel like it.Exceptions: minors, enemy aliens

Page 11: INTRODUCTION - University of Victoria - Web.UVic.caweb.uvic.ca/~lssweb/wp-content/outline_uploads/76... · Web viewDow Corning (2000) (BCCA) Facts: Class actions for negligence in

Corporations: extra-provincial companies (not incorporated in BC) have two months to register in BC in order to bring action. If not registered, they may not be able to bring action (unclear).

Bumper Development Corp Ltd Case (1991) (Eng. C.A.)Facts: Revolves around religious artifacts that were found in India buried near a temple. The artifacts were sold and brought to the United Kingdom where Bumper Development (an Alberta company) attempted to buy them (in good faith). Bumper claimed to buy the artifacts in order to donate them to a museum. When UK authorities learned of the artifacts, they seized them. Bumper responded by suing the UK government. During the litigation, several claimants joined the action claiming rights to the artifact, including the “temple” (brought by an individual on behalf of a physical place).

Issues:Does the particular party have status or recognition as a legal person in the country where it exists (jurisdic entity under the law of that country)If have JE, would the forum accept that (an issue of comity)

Definition of jurisdic entity:“A person, body of persons or object who or which is recognized by the law concerned as being capable of enjoying legal possession of or title to an object an of suing or being sued in respect thereof.”

When one jurisdiction is applying the law of another jurisdiction, the court itself cannot simply enforce that law. The foreign law is treated as a question of fact which must be proved in evidence.

Court can question or raise issues with experts, cannot conduct and rely on its own research into foreign lawIn the absence of any contrary evidence, assumed same as English lawIf there is conflicting evidence, court is bound to look at sources to decide

Held: Under Hindu law the temple was a juristic entity and the third claimant had the right to sue and be sued on behalf of the temple.

Court found that different jurisdictions take different views – used examples of Roman Catholic Cathedrals in France which need to protect their assets in other places.In Indian law, the temple would in fact be able to act as a plaintiff if the artefacts were still in India.

Lex forum – the law of the forum determines who can bring action:In this case England, where the temple could not bring action - not a corporation, person or registered society, rather a thing.However, Court relied on its inherent jurisdiction to control its own procedure, including who can be a plaintiff. Decided that since the temple would have had the right to bring action in India, that they should respect that right and allow the temple to act as a plaintiff in England.

Recognize in this case according to principles of comity

Page 12: INTRODUCTION - University of Victoria - Web.UVic.caweb.uvic.ca/~lssweb/wp-content/outline_uploads/76... · Web viewDow Corning (2000) (BCCA) Facts: Class actions for negligence in

Temple had a vested right in India to bring action and carried this right with it to England

Result: Bumper lost and the artefacts were returned to India. Plaintiff also claimed damages against Bumper.

Jurisdiction over the Defendant: Within the Province If the defendant is in the province, then there is no conflicts issue.

Territoriality: at CL, provinces have jurisdiction over any person within their territory Jurisdiction as of right (jurisdiction simpliciter): if you can serve documents to a person in the province, then the court has jurisdiction.

Exceptions: entities that cannot be sued in Canada:The Crown except through the Crown Proceedings ActForeign governments, unless engaged in commercial activityForeign diplomats (diplomatic immunity)

Wrinkle - concept of the fleeting presence:

Maharanee of Baroda v. Wildenstein (1972) (Eng. C.A.) Facts: Both parties lived in France but are occasionally in England. M bought a painting from W in France and it was subsequently discovered to be a fake. Issue: does the English court have jurisdiction over the foreign defendant?Held: fleeting presence is sufficient

Writ was properly served on W while he was in England – therefore validIrrelevant that the plaintiff and defendant are foreigners, as long as both in England at time action commenced.

BUT concept of fleeting presence overruled by s. 3 of the BC Court Jurisdiction and Proceedings Transfer Act.

S. 3 - A court has jurisdiction simpliciter only if the person has ordinary residence in BC – cannot bring action where a person only has fleeting presence. However, court will also have jurisdiction in the following circumstances.

If you’re a plaintiff in another claim in the province, considered to have ordinary residenceIf you show up during the proceeding, you submit to the jurisdictionMay be a clause in a contract explicitly stating that a party will submit to a particular court jurisdictionIf there is a real and substantial connection between BC and the facts on which the proceedings against the defendant is based.

Court Jurisdiction and Proceedings Transfer ActDerives from the Uniform Law Conference of Canada – attempt to achieve consistency across the provinces on when a court will have jurisdiction, and when they will enforce foreign court ordersPremises on Morguard and Hunt – largely based on idea of real and substantial connection

Page 13: INTRODUCTION - University of Victoria - Web.UVic.caweb.uvic.ca/~lssweb/wp-content/outline_uploads/76... · Web viewDow Corning (2000) (BCCA) Facts: Class actions for negligence in

Anticipated that other provinces will enact similar legislation. Three parts:

Part I: DefinitionsPart II: Territorial competence of the courts of BCPart III: Attempt to create a system by which proceedings will be transferred among provinces, rather than having to start law suit anew (not in materials).

Relocates the issue of whether the court has jurisdiction into the legislation – moves it away from the Rules of Court. Start with the Act, but must still consult the Rules of Court when they discuss service ex juris.

Jurisdiction over the Defendant: Outside the ProvinceMuch of conflict of laws is dealt with through the rules of court, many of which deal with foreign defendants.

Service ex juris – serving a defendant outside of the court’s jurisdiction. Upon receipt of the documents, the defendant is then subject to the court’s jurisdictionCommon law does not provide for service ex juris – rather must be statutorily based, usually in the Rules of Court.

British courts used to require an ex parte hearing before there could be service ex juris, for reasons of territoriality.

Courts are not always directly involved in service ex juris – rather there is a presumption that you can serve your documents in a certain way. However, the courts can always control this process.

Threshold question: jurisdiction simpliciterImportant first step in commencing an action against a foreign defendant - not generally difficult to establishCourt Jurisdiction Act now refers to this as “territorial competence” (slightly different but essentially equivalent to jurisdiction simpliciter). Until 2006, it was the BC Rules of Court that dealt exclusively with BC court jurisdiction over foreign defendants.

Key rule is Rule 13(1): still the operative civil procedural rule, but is now supplemented by the Court Jurisdiction Act.

Intent of the CJA in this context is to codify and clarify the existing principles respecting foreign entities and facts and nature of the connection with BC courts that is required. Previously, was done implicitly through the Rules of Court – rules regarding service ex juris remain in place, however, they now operate in conjunction with the CJA.Many of the old cases (but not all) on service ex juris remain important in understanding the legislation, particularly for their procedural aspects.

Service ex juris:Rule 13(1) allow for service ex juris without court involvement in any of the situations that are explicitly set out in CJA s. 10In cases where the facts do not fit within Rule 13(1), court has discretion to allow service ex juris – requires an order from the court.

To get this kind of court order, you must fit within CJA s. 3(e) – real and substantial connection.

Page 14: INTRODUCTION - University of Victoria - Web.UVic.caweb.uvic.ca/~lssweb/wp-content/outline_uploads/76... · Web viewDow Corning (2000) (BCCA) Facts: Class actions for negligence in

Note: must also watch CJA s. 6 - service ex juris without a real and substantial connection

Establishing a Real and Substantial ConnectionReal and substantial connection is caught somewhat generically by CJA s. 3(e): it is a real and substantial connection between BC and the FACTS that gives the court jurisdiction

But what are the “facts”?CJA s. 10 – if your facts fit within these situations, there is a presumption of a real and substantial connection

Not exhaustive, but helpful to fall within these situationsE.g. if the tort is committed in BC

“Real and substantial connection” is always arguable – no clear definition, but rather a series of considerations

Constitutionally-based. Derives from:Comity owed by one province to other provincesInherent limitations on the jurisdiction of the provincial courts in the administration of justice

Challenging Court JurisdictionDefendant can challenge the jurisdiction (territorial competence) of the BC court under Rule 14(6)

Rule 6: If you are coming to court to challenge jurisdiction simpliciter, this does NOT mean that you are submitting to the court’s jurisdiction.

Oakley v. Barry (1998) (NSCA)Facts: Plaintiff is resident in Nova Scotia, brings an action in the Nova Scotia courts against physicians at a hospital in New Brunswick. Claims relate to damages caused by a misdiagnosis – that she had infectious hepatitis B. Took place while the plaintiff was living in New Brunswick.

Service ex juris was authorized by the Nova Scotia Civil Procedure Rules: court had jurisdiction if the defendant could be served physically in Canada or the United States. (Different than the rules in BC, now and in the past).

Argument: Defendant challenged the action on the grounds that the Nova Scotia court did not have jurisdiction simpliciter. Plaintiff argued that there was jurisdiction because she had received significant medical care and support, and discovery of the misdiagnosis took place, in Nova Scotia due to the diagnosis. Although the Civil Procedure Rules appeared to give jurisdiction, court still has the authority to control its procedure.

Reasoning: Applied Morguard - jurisdiction must accord with order and fairness, which is tied to RSC

Criteria must be applied in a flexible manner. In this case, there was a real and substantial connection between:

The subject matter of the action and the province of Nova Scotia (limited connection)

Page 15: INTRODUCTION - University of Victoria - Web.UVic.caweb.uvic.ca/~lssweb/wp-content/outline_uploads/76... · Web viewDow Corning (2000) (BCCA) Facts: Class actions for negligence in

The damages caused suffered by the plaintiff and the province of Nova Scotia.

However, no defendant connection to Nova Scotia

Considered a number of factors (none of which are determinative):Nova Scotia is responsible for providing care to the plaintiffLook at fairness to the plaintiff – incapable of bringing action anywhere but in Nova Scotia.

Would be unfair to deny claim because otherwise could not bring actionSelected Nova Scotia because that’s where she lives – did not raise concerns of forum shoppingBriefly discussed forum non conveniens – whether the court should exercise its jurisdiction

Held: Nova Scotia should be able to serve as the forum although the subject matter had only limited connection with NS, and the defendants had none.

Turbide – serves as a counter example to OakleyFacts: BC resident brought and action against doctors in Nova Scotia. There had been a motor vehicle accident in Nova Scotia. Plaintiff was misdiagnosed by Nova Scotia doctors. Returns to BC where discovers the misdiagnosis and wants to bring action against Nova Scotia.

Held: Leave was denied by the BC courts:Damages accrued in Nova ScotiaNothing happened in BC – only connection is that the Plaintiff resides in BC.

Gauthier (NBCA)Facts:

Plaintiff is the widow of the deceased, who was killed in a car accident. Plaintiff lives in New Brunswick. Here husband had “ordinarily” lived in New Brunswick. The defendant, the diver (also deceased), was ordinarily resident in Quebec (presume that the insurance was also from Quebec). The accident took place in Ontario, while the deceased was on his way home to New Brunswick. Both of the deceased knew each other and had worked together, but had not close relationship. Plaintiff brought an action against the defendant in New Brunswick.

Reasoning: Court is very careful to distinguish between jurisdiction simpliciter and forum conveniens – have very different consequences.

Discuss Morguard and Hunt – based on order and fairness to the litigants, met by the RSC test.

Para. 68 – may be situations where there may be multiple jurisdiction who have a real and substantial connection sufficient to meet jurisdiction simpliciter, but may not be the one with the real and substantial connection (forum non conveniens).

Page 16: INTRODUCTION - University of Victoria - Web.UVic.caweb.uvic.ca/~lssweb/wp-content/outline_uploads/76... · Web viewDow Corning (2000) (BCCA) Facts: Class actions for negligence in

RSC test focuses on the connection of the subject matter to the province.

In this case,Defendant has no connection to New BrunswickBut the damages were suffered in New Brunswick, providing a real and substantial connectionThere was no real question that the driver was negligent. Real case was the quantum of damages. The quantum of damages was experienced in New Brunswick. Refers to Quebec rules of court, which says that there would be a RSC to Quebec if the damages were suffered in QuebecDiscusses the unfairness and costs to the plaintiff if they had to bring action elsewhere (not determinative)In tort cases, there is a predominant concern about where the tort was committedRelationship between the two deceased: court made much of the fact that the defendant knew, or would have known, that as a result of their negligence that the damages would be suffered in New Brunswick (mourning wife and children), this helped create a real and substantial connection.

CJA Section 10CJA s. 10: presumed connection between the facts and the BC court. Mirrors to a point what was in the old Rule 13(1).

10 Without limiting the right of the plaintiff to prove other circumstances that constitute a real and substantial connection between British Columbia and the facts on which a proceeding is based, a real and substantial connection between British Columbia and those facts is presumed to exist if the proceeding:

(a) is brought to enforce, assert, declare or determine proprietary or possessory rights or a security interest in property in British Columbia that is immovable or movable property, (b) concerns the administration of the estate of a deceased person in relation to

(i) immovable property in British Columbia of the deceased person, or (ii) movable property anywhere of the deceased person if at the time of death he or she was ordinarily resident in British Columbia,

(c) is brought to interpret, rectify, set aside or enforce any deed, will, contract or other instrument in relation to

(i) property in British Columbia that is immovable or movable property, or (ii) movable property anywhere of a deceased person who at the time of death was ordinarily resident in British Columbia,

(d) is brought against a trustee in relation to the carrying out of a trust in any of the following circumstances:

(i) the trust assets include property in British Columbia that is immovable or movable property and the relief claimed is only as to that property; (ii) that trustee is ordinarily resident in British Columbia;

Page 17: INTRODUCTION - University of Victoria - Web.UVic.caweb.uvic.ca/~lssweb/wp-content/outline_uploads/76... · Web viewDow Corning (2000) (BCCA) Facts: Class actions for negligence in

(iii) the administration of the trust is principally carried on in British Columbia; (iv) by the express terms of a trust document, the trust is governed by the law of British Columbia,

(e) concerns contractual obligations, and (i) the contractual obligations, to a substantial extent, were to be performed in British Columbia, (ii) by its express terms, the contract is governed by the law of British Columbia, or (iii) the contract

(A) is for the purchase of property, services or both, for use other than in the course of the purchaser's trade or profession, and (B) resulted from a solicitation of business in British Columbia by or on behalf of the seller,

(f) concerns restitutionary obligations that, to a substantial extent, arose in British Columbia, (g) concerns a tort committed in British Columbia, (h) concerns a business carried on in British Columbia,

(i) is a claim for an injunction ordering a party to do or refrain from doing anything (i) in British Columbia, or (ii) in relation to property in British Columbia that is immovable or movable property,

(j) is for a determination of the personal status or capacity of a person who is ordinarily resident in British Columbia, (k) is for enforcement of a judgment of a court made in or outside British Columbia or an arbitral award made in or outside British Columbia, or (l) is for the recovery of taxes or other indebtedness and is brought by the government of British Columbia or by a local authority in British Columbia.

Torts Occurring Within BC – CJA 10(e)In dealing just with tort, there is some flexibility for service ex juris, but some elements of fairness are helpful. The idea is that the plaintiff must be able to bring a good arguable case.

In BC, not just a question of where the act took place appears to be an element of intent, especially in a products liability context. In car accidents, its more difficult to manufacture that element of intent, but reasonable to contemplate that you might to collide with someone from outside of the jurisdiction.However, can always turn to CJA s. 3(e): real and substantial connection. For thoroughness, you should always plead both.

Moran v. Pyle (1973) (SCC)Facts: Saskatchewan resident gets fatally electrocuted while replacing a light bulb. Family brought suit in Saskatchewan against the manufacturer for negligence. The manufacturer was an Ontario company with no assets or offices in Saskatchewan. Bulbs were sold into Saskatchewan through various intermediaries. Saskatchewan’s

Page 18: INTRODUCTION - University of Victoria - Web.UVic.caweb.uvic.ca/~lssweb/wp-content/outline_uploads/76... · Web viewDow Corning (2000) (BCCA) Facts: Class actions for negligence in

Queens Bench Act prohibited bringing action for a tort committed outside the province without the leave of the court.

Issue: Where was the tort committed? Trial: tort occurred in Ontario – shoddy craftsmanship during manufacture, but allowed leave to bring action.Saskatchewan Court of Appeal: reversed the decision

Reasoning: Situs of the court for jurisdiction simpliciter is different from the situs of the tort for choice of law (covered later). Just because the court has jurisdiction simpliciter and chooses to exercise this discretion, doesn’t mean that the law of the forum applies (lex fori).

For purposes of jurisdiction simpliciter, SCC discusses two different theories:Place of acting theory: tort occurs where the act is done that ultimately results in the harm – in this case would be Ontario, where the light bulb was negligently made

Rejected as too rigid – there is no tort until the product malfunctions.Place of harm theory: the last necessary element it the ground for where the tort occurs – in this case would be Saskatchewan, where installing light bulb was the last necessary element.

Rejected as problematic for the purposes of jurisdiction simpliciter.The court rejects both of these approaches:

“Generally speaking, in determining where a tort has been committed, it is unnecessary and unwise to resort to any arbitrary sort of rules. The place of acting and the place of harm theories are too arbitrary and inflexible to be recognized in contemporary jurisprudence.” 

Supports a rule that is much more principled in nature: Real and substantial connection will attach mostly to the effects, and it would not be inappropriate to “regard a tort as having occurred in any country substantially affected by the defendant's activities or its consequences and the law of which is likely to have been in the reasonable contemplation of the parties.” 

Rule: “Where a foreign defendant carelessly manufactures a product in a foreign jurisdiction which enters into the normal channels of trade and he knows or ought to know both that as a result of his carelessness a consumer may well be injured and it is reasonably foreseeable that the product would be used or consumed where the plaintiff used or consumed it, then the forum in which the plaintiff suffered damage is entitled to exercise judicial jurisdiction over that foreign defendant.  

Recognizes the important interest a state has in injuries suffered by persons within its territory.  Purpose of negligence as a tort is to protect against carelessly inflicted injury and thus that the predominating element is damage suffered.

Held: Tort occurred in Saskatchewan and was reasonably within the contemplation of the manufacturer. Doesn’t necessarily mean that the law of Saskatchewan would occur in this case, but does mean that Saskatchewan has jurisdiction simpliciter.

Page 19: INTRODUCTION - University of Victoria - Web.UVic.caweb.uvic.ca/~lssweb/wp-content/outline_uploads/76... · Web viewDow Corning (2000) (BCCA) Facts: Class actions for negligence in

By tendering his products in the market place directly or through normal distributive channels, a manufacturer ought to assume the burden of defending those products wherever they cause harm as long as the forum into which the manufacturer is taken is one that he reasonably ought to have had in his contemplation when he so tendered his goods.  This is particularly true of dangerously defective goods placed in the interprovincial flow of commerce. 

Furlan v. Shell Oil (2002) (BCCA)Facts: Plaintiff (class action law suit) alleged that harm was caused by polybutylene plumbing system. Three defendants: all companies that manufacture the plumbing resin.

Defendants: none carried on business in BC, were not the manufacturers of the plumbing equipment (were quite far removed from the damaged) and had reasonable defences.

Arguments: Shell argued that they were not selling resin directly to Canada, but didn’t deny that it might have showed up here.Second company: did not admit to selling resin to Canada and didn’t know how it showed up hereDuPont: admitted to selling resin into Quebec, but not to any of the companies that were involved in the law suit.

Issue: Sufficiency of the evidence to support the service ex juris and the meeting of the old Rule 13(1). Here the plaintiff has the burden of proof and the defendant can challenge that under Rule 14(6).

Defendant companies challenged service ex juris under Rule 14(6) – argued that the plaintiffs had not made out their case and the pleadings were not sufficient to support service ex juris

I.e. argued that the plaintiff had not disclosed evidence of a connection between the harm suffered and the resin sold by the companies (or at least a company)

Here the defendants did not provide evidence to negate the allegations.

Reasoning: Jurisdiction simpliciter is different than forum non conveniens: if the case is made and you fit within the pattern, then JS is there.

Basic rules to determine jurisdiction simpliciter:Basic rule: jurisdiction simpliciter is determined based on pleadings.

A case does not have to be made out on the evidence where the facts of the pleadings are not challengedIf the defendant does challenge the pleadings with evidence, the plaintiff must respond with evidenceIf the defendant counters by affidavit evidence, there is an obligation on the plaintiff to provide evidence to counter or explain the defendant’s affidavit

Residual category (qualification on the proposition that JS is to be determined by the pleadings): Where plaintiff puts forward an extremely tenuous

Page 20: INTRODUCTION - University of Victoria - Web.UVic.caweb.uvic.ca/~lssweb/wp-content/outline_uploads/76... · Web viewDow Corning (2000) (BCCA) Facts: Class actions for negligence in

claim, affidavit evidence would be needed to support it concern as to whether there is ANY evidence to support the claim

This would be necessary where application is made for ex juris service and the claim is extremely tenuousIf plaintiff cannot make a good arguable case on any material fact that is crucial to the case, jurisdiction should be refused (otherwise risk abuse of process) Useful on questionable suits that are begun strictly on pleadings.

Held: BC courts have jurisdiction simpliciter service ex juris is allowed.

Muscutt v. Courcelles (2002) (Ont. CA)Facts: Car accident. Plaintiff was a passenger in a vehicle driven by one defendant, owned by another defendant, struck by the defendant driver of an ambulance. Accident occurred in Alberta. Plaintiff suffered serious spinal cord injuries. Returned to his home province of Ontario and commenced an action there.

Clear that the tort occurred in Alberta.

Issue: Was this “damage sustained in Ontario”? Was this principle an inappropriate extension of their jurisdiction?

Different to Moran since the ongoing suffering was taking place in Ontario, not the initial accidentDefendant argued that action should be stayed for want of jurisdiction, since the plaintiff’s pain and suffering in Ontario did not amount to a RSC to do so would have an inappropriate and disproportionate effect on the defendant

Reasoning: Key consideration is the content of real and substantial connection. The court considered to different approaches for determining real and substantial connection:

Personal subjection approach (the older approach): Focuses attention on the defendant and the connection between the defendant and their actions which would make it reasonable for them to contemplate that they might be sued in the Province of BC. This idea raised in Moran v. Pyle – reasonable contemplation of the defendant (sort of)

Administration of justice approach (the newer approach): Broader than personal jurisdiction approach. All that is required to establish a RSC is that there is a minimum standard of suitability: rests in part on the fairness of the case in the forum in which it is being brought.

Ontario prefers the broad approach SCC has made it clear that RSC is flexible and should be flexibleSCC, at least in interprovincial cases, ensures a high degree of respect about the recognition and enforcement of foreign judgements: for order and fairnessBroad approach is the product of the relationship between JS and forum non conveniens – encourages broad approach

The real and substantial connection test involves a fact-specific inquiry, but the test ultimately rests upon legal principles of general application.  Asking two separate questions:

Page 21: INTRODUCTION - University of Victoria - Web.UVic.caweb.uvic.ca/~lssweb/wp-content/outline_uploads/76... · Web viewDow Corning (2000) (BCCA) Facts: Class actions for negligence in

(1) jurisdiction simpliciter – is the minimum threshold there? Under jurisdiction simpliciter, you are not looking for the most convenient forum, just the minimum standards of whether R&S has been met.“The question is whether the forum can assume jurisdiction over the claims of plaintiffs in general against defendants in general given the sort of relationship between the case, the parties and the forum.” 

(2) forum non conveniens – is this is the most suitable place to hear the case?

Under forum non conveniens, you are looking for the most suitable forum as a matter of discretion“By contrast, the forum non conveniens test is a discretionary test that focuses upon the particular facts of the parties and the case.  The question is whether the forum should assert jurisdiction at the suit of this particular plaintiff against this particular defendant.”

There is some overlap between jurisdiction simpliciter and forum non conveniens - many of the same factors are considered

Overlapping considerations include the connection of the forum to the plaintiff and defendant, fairness to plaintiff and defendant

Factors that are relevant in the administration of justice approach:Connection between the forum and the plaintiffs claim

Forum has a legal interest in protecting its residents – mere residence is not sufficient, but notes that the medical costs are falling in OntarioImportant interest a state has in injuries suffered within its territoryStretches Moran case which looked at the contemplation of the parties (if you produce products that are defective, you should contemplate it would cause injury in a number of places)

Connection between the forum and the defendantIf D has done anything within that jurisdiction that bears upon the claim advanced by the P, then case for jurisdiction would be strongerIn this case, D had no connection with Ontario, but there is an inherent risk that driving around Alberta you might hit someone from Ontario.

Unfairness to the defendant in assuming jurisdictionVery much like forum non conveniens considerationsOne of the key points that persuaded the court. Court emphasized fact that it was the insurance industry that was defending the action. There is really no unfairness to the insurance companies - automobile insurers are national companies that can handle the litigation anywhere in CanadaUniformity in terms of insurance – prospect of this going inter-provincially. Not at the international level – insurance schemes would not necessarily be similar.

Unfairness to the plaintiff in NOT assuming jurisdictionRelies on Morguard: need to consider the plaintiffs interest in access to the courts and the inconvenience of compelling them to bring and action in Alberta.

Involvement of other parties in the suitAvoiding risk of inconsistent results and a multiplicity of suits, especially

Page 22: INTRODUCTION - University of Victoria - Web.UVic.caweb.uvic.ca/~lssweb/wp-content/outline_uploads/76... · Web viewDow Corning (2000) (BCCA) Facts: Class actions for negligence in

parties outside the jurisdictionAgain, very much like a forum non conveniens consideration Not an issue in this case

Reciprocity (Court’s willingness to recognize and enforce extra-provincial judgements that were rendered on the same jurisdictional basis)

Want to consider the grounds on which the another court might have exercised jurisdiction“Every time a court assumes jurisdiction in favour of a domestic plaintiff, the court establishes a standard that will be used to force domestic defendants who are sued elsewhere to attorn to the jurisdiction of the foreign court or face enforcement of a default judgment against them.” Cases of an interprovincial or international nature will be handled differentlyOntario judgment would be enforced elsewhere, but also need to contemplate a reverse of the action i.e. domestic D sued in “sister provinces”

If international or interprovincial nature: Morguard, Tollefson & Hunt Assumption of jurisdiction more justified in interprovincial cases as opposed to international cases because of the context of the Canadian federation and sister provincesAssumption of jurisdiction not as easy in international environment

ComityShould consider the standards of jurisdiction, recognition and enforcement In interprovincial cases, same standard applies within Canada; more complex for international casesShould probably be considered under forum non conveniens

Held: Jurisdiction simpliciter had been met.

Sample Ontario cases where there was no jurisdiction simpliciter:Plaintiff is injured in a motor vehicle accident in New York StatePlaintiff injured in a fall in a restaurant in Buffalo NYPlaintiff falls while rappelling during a package holiday in Costa Rica

Applying the 8 Considerations:1) All have a connection between the plaintiff and the action2) None have connection between forum and the defendant 3) Would be unfair to the defendant to force them come to Ontario, even in the

case of New York4) Unfairness to the plaintiff: not unfair to require the plaintiff to bring the action

where the claim arose. 5) Involvement of other parties – not an issue6) Willingness to recognize extra-provincial judgements – wouldn’t recognize and

enforce judgements with such a tenuous connection7) Doesn’t really apply8) Doesn’t really apply

Page 23: INTRODUCTION - University of Victoria - Web.UVic.caweb.uvic.ca/~lssweb/wp-content/outline_uploads/76... · Web viewDow Corning (2000) (BCCA) Facts: Class actions for negligence in

Armeno Mines v. Pukuafu Indah (2000) (BCCA) Facts: Contract to acquire share in proposed mining development in Indonesia not carried out. The contract had been made in BC and was to be subject to BC law.

Plaintiff argued that BC court had jurisdiction under Rule 13(1)(h) – tort committed within BC, which includes breach of contract. Trial judge set aside the ex juris service because had no jurisdiction over the defendant.

Rule: Where there is a tenuous claim, jurisdiction simpliciter cannot be decided on pleadings alone will have to look to evidence to make out a good arguable case

Tenuous = evidence introduced by the foreign defendant contradicts material facts pleaded by the plaintiff, or otherwise proves fatal to the plaintiff’s claimGood arguable case = serious question to be tried, genuine issue, or with some chance of success not a high evidentiary threshold

Plaintiff does not have to tender evidence about issues that the defendant’s evidence has not put at issue

Held: Although the plaintiff made a case for jurisdiction simpliciter on the pleadings alone, refused jurisdiction on the grounds that they had failed to make a good arguable case on the evidence

In the pleadings, the plaintiff claimed to be “substantially affected” by the breach in BC and pointed to the fact that the contract was made in BCHowever, the plaintiff hadn’t made out a good arguable case on the evidence - hadn’t established causation from inducementPlaintiff was required to make some evidentiary response to the defendant’s affidavits sufficient to raise a triable issue couldn’t rely on pleadings alone.

Defamation – Complex!Bangoura v. Washington Post (2005) (OCA)Fact: Plaintiff brought action, primarily in defamation, in Ontario against the Washington Post and 3 of its journalist about two articles in the Post in 1997 which talked about the Plaintiff’s activities in the Ivory Coast. Plaintiff had worked for the UN for drug control, was dismissed, and the Post alleged that he had engaged in disreputable conduct. Plaintiff then moved to Ontario. Commenced action in the Ontario courts in 2003.

Only 7 copies of the Washington Post featuring the story had been sold in OntarioAlthough the story was also online for 14 days (then went to the archive (paid) website), only 1 person had viewed the article online.

Trial: Found that Ontario court had jurisdiction simpliciter found that newspaper was internationally known and read and that there was no question that it could disadvantage the plaintiff.

Page 24: INTRODUCTION - University of Victoria - Web.UVic.caweb.uvic.ca/~lssweb/wp-content/outline_uploads/76... · Web viewDow Corning (2000) (BCCA) Facts: Class actions for negligence in

Court of Appeal: reversed the decision, saying that there was no real and substantial connection to the forum. Hence, no jurisdiction simpliciter.

When the tort arose, the plaintiff was nowhere near Ontario. Damages occurred primarily in the Ivory Coast. No significant connection between the defendant and Ontario (minimal connection from publication). Unfairness to the defendant to force them to come to Ontario.

Burke v. The New York PostFacts: Burke was the GM of the Vancouver Canucks. Post wrote uncomplimentary things about Burke in a news article. No copies of the paper were sold in BC. Post does have a website, but no indication of how many people accessed the website. But a BC talk show brought up the article and then confronted Burke about it.

New York Post applied for a stay of proceeding, arguing that there was no jurisdiction simpliciter.

Held: Found that BC had jurisdiction simpliciter real and substantial connection to BC since foreseeable on the part of the defendant that the story available online about a businessman with a strong connection to BC, might actually be read by people in BC.

This was the connection: foreseeability of the story being read in BC.

Contractual Obligations - Court Jurisdiction Act s. 10(e)Broader than the old provisions:

(e) concerns contractual obligations, and (i) the contractual obligations, to a substantial extent, were to be performed in British Columbia, (ii) by its express terms, the contract is governed by the law of British Columbia, or (iii) the contract

(A) is for the purchase of property, services or both, for use other than in the course of the purchaser's trade or profession, and (B) resulted from a solicitation of business in British Columbia by or on behalf of the seller,

CJA s. 10(e)(i) considers primarily performance of contract: Appears to exclude contracts that were simply made (but not performed) in BC. Further, if the contract is breached in BC, but wasn’t supposed to be performed here, likely no connection. Draws distinction between the formalities of the contract and the performance of the contract.

Also note CJA s. 10(h) - concerns a business carried on in British Columbia,

US Satellite Broadcasting v. WCI TV (2001) (ABCA)Facts: WIC had exclusive right for Canadian satellite service. US companies provided satellite decoders to Canadians. WIC sued the US companies for interfering

Page 25: INTRODUCTION - University of Victoria - Web.UVic.caweb.uvic.ca/~lssweb/wp-content/outline_uploads/76... · Web viewDow Corning (2000) (BCCA) Facts: Class actions for negligence in

with its statutory exclusive right to provide satellite service. Defendants provided affidavit evidence that when they learned they were providing the signal to a Canadian they would cut it off.

Canadian plaintiffs had gone to California in an attempt to bring action against a number of American companies. US Federal Court judge turned the case down, stating that Canada was a more appropriate forum to bring the action, since all the damages were suffered in Canada.

Conditional dismissal in California – the action was stayed on the condition that the US defendants had to show up for court in BC. If the defendants didn’t show up in BC, then they could bring the case back to California.

Held: Court applied the real and substantial connection test. Stated that the plaintiff had a good arguable case: “a proper one to be heard in our courts”. Also stated that the onus for establishing service ex juris is lax – only need prima facie evidence.

Three different American defendants:US Satellite Broadcasting – company based in Minnesota provided satellite TV in the US; no presence in Canada. They did not deny that their broadcast signal went into Canada. Admitted to activating decoders in Canada, and to receiving payment for these decoders. Indicated that they would discontinue any service that was being received in Canada.

Court found that there was damages done to the Canadian company USSB essentially carrying on business in Canada = jurisdiction simpliciter

Echo Star – No presence in Canada. Decoders were readily available in Manitoba and BC, but not in Alberta where the action was brought. Argued no connection to Alberta in particular.

Court found jurisdiction simpliciter anyway WIC had a Western Canada license and the damage to the business itself was occurring in Alberta (dubious connection).

Warren: Provided satellite equipment and had sold to Canadian companies. No presence in Canada.

Jurisdiction simpliciter granted.

Comment: The fact that California had kicked out the case out for most of the defendants likely played a role in having Alberta courts accept the case. Sort of like S. 6 – jurisdiction of last resort.

Purple Echo v. KCTSFacts: Plaintiff claims damages for breach of contract. Defendant is a public broadcaster in Seattle. Plaintiff argues that KCTS did not properly show a series of 13 TV programs as required under contract – didn’t do enough to promote or show them at a reasonable time. Documents served in Seattle without leave of the court, relying on Rule 13(1).

Page 26: INTRODUCTION - University of Victoria - Web.UVic.caweb.uvic.ca/~lssweb/wp-content/outline_uploads/76... · Web viewDow Corning (2000) (BCCA) Facts: Class actions for negligence in

KCTS argued that the BC courts did not have jurisdiction simpliciter.

Held: Plaintiff attempted to establish jurisdiction simpliciter based on three arguments:

Under CJA s. 3(e) – real and substantial connection:Tried to establish that KCTS had a place of business within BC, making them subject to BC jurisdiction. An organization in BC called PCPTA collected donations on behalf of KCTS in BC.Court found that although PCPTA did have connection with KCTS, that it was really a contractual association and that the PCPTA was not KCTS. Thus, did not fall within CJA s. 3.

Under CJA s. 10(e)(i) – contractual obligations to be performed in BC:Court did a detailed examination of the contract and it obligations. Contract said that KCTS was under no obligation to actually air the series. Licensed to broadcast only in the US, but broadcast available through cable and satellite in BC. Much of KCTS funding comes from BC. Claims regarding the breach of contract: almost all of the claimed obligations were to be performed in Seattle. Nothing in the contract said that the breaches of contract occurred in BC.

Focused on the allegations of the breach rather than on the contract as a whole.

Thus, did not fall within CJA s. 10(e)(1).Under CJA s. 10(h) - concerns of business carried on in BC

Para 38-41: JS applies to ANY business carried on in BC. Although plaintiff clearly carries out business in BC, Court decides that the law focuses on the connection between the court and the defendant. Based on this approach, had already found that KCTS was not carrying on business in BC (question 1) Thus, did not fall within CJA s. 10(h).

Residual DiscretionCJA s. 6: A court that under section 3 lacks territorial competence in a proceeding may hear the proceeding despite that section if it considers that

(a) there is no court outside British Columbia in which the plaintiff can commence the proceeding, or (b) the commencement of the proceeding in a court outside British Columbia cannot reasonably be required.

Under residual discretion, a BC court can have JS without a real and substantial connection!

S. 6 is essentially a clause of last resort to establish JS. However, may be constitutionally suspect because of unfairness to the defendant.Generally, it is the real and substantial connection that ensures order and fairness. Without such a connection, may be problematic. Section will likely be interpreted narrowly.Plaintiff will have significant burden in demonstrating that no other court can commence proceedings.

Plaintiff may need to show that they have been shut out of other courts.

Page 27: INTRODUCTION - University of Victoria - Web.UVic.caweb.uvic.ca/~lssweb/wp-content/outline_uploads/76... · Web viewDow Corning (2000) (BCCA) Facts: Class actions for negligence in

Class Action SuitsIn conflicts, class actions suits are always treated a little bit different.

Harrington v. Dow Corning (2000) (BCCA)Facts: Class actions for negligence in manufacturing silicone breast implants

BC Class Actions Proceedings Act s. 16(2) does appear to allow someone who is not a resident in BC to be involved in a class action in BC.

Only BC and Ontario have Class Action statutesIssue: whether JS existed for those persons who had opted into the class action but who were not residents of BC. I.e. does the Class Proceedings Act permit the court to take wider jurisdiction than it otherwise would? If yes, it is unconstitutional?

Allows a non-resident to opt into a class action lawsuit

Defendant argued that people in the class action must have some connection to BC. Normally, Court would not have jurisdiction over non-resident claims unless there was a real and substantial connection to BC.

However, class action suits add an extra dimension

Held: Trial judge and BCCA both found that there JS over non-residents who wanted to opt into the class action law suit.

Anchored in fundamental values of fairness: orderly decision making comity

“Do not mean to suggest that a court may assume jurisdiction at a plaintiff's request for her convenience. More than a plaintiff's choice is required. I do suggest that the existence of a certified class action may be that something more. It may, depending on the nature of the cause of action and the certified common issues, provide a sufficient connection to justify a claim to jurisdiction. So long as the process is fair, there need be little concern at this stage for the interests of a defendant; they are well protected by the doctrine of forum non conveniens.”

Within context of class action, convenience, practicality and mobility of people supports bringing people in to suitable locations (BC or ON) but this must be balanced against identified issues which in the Canadian context raises constitutional concerns of provincial legislation that includes non-residents. If there was a sufficient number in province, then it could be said that bringing in other people would be incidental, but if no residents from BC, then can't be said that non-BC residents are incidental.

Note: might it have been different if dealing with an “opt-out” act Comment: R&C is here more because of the nature of the action – class action changes the action of JS.

Page 28: INTRODUCTION - University of Victoria - Web.UVic.caweb.uvic.ca/~lssweb/wp-content/outline_uploads/76... · Web viewDow Corning (2000) (BCCA) Facts: Class actions for negligence in

Forum Non ConveniensAnti-suit injunctions: injunction granted in one jurisdiction to prevent party from proceeding with an action in a second jurisdiction (restraint exercised over the litigants, NOT over the foreign court!)

If you are in one jurisdiction and try to move your case to another jurisdiction, you will be held in contempt in court in the current jurisdiction. Prevents you from hopping to another court.Rarely grantedPrinciples:

Granted where the ends of justice require it Injunction is directed at the individuals, not the foreign courtWill only be granted where the party seeking it is amenable to the jurisdiction of the court granting it (i.e. only going to issue where it can be effective)

When one looks at these processes, have two aspects to consider:Stay of proceedings

Court asked to stay proceedings that have been brought in that jurisdiction on the basis on the notion of the D, that the jurisdiction is not the more suitable.Ordinarily a D’s motion, and has that burden to meet

Anti-suit injunction Court saying persons over whom it has control are not permitted to litigate there. Much cleaner and more consistent with comity if went to foreign jurisdiction and ask foreign court to stay its decision. But doesn’t always happen this way Even though is not an injunction against foreign Court, indirectly says the foreign court is not suitable have an immediate and obvious conflict

Forum non conveniens is really a question of how much deference should be given to a plaintiff’s choice of forum. Three issues:

What is the standard to be used by courts in declining to exercise jurisdiction?

Influenced by many different factors:Juridical advantage: what benefit is the plaintiff getting from bringing action in one jurisdiction that they would be declined in another jurisdiction? Who has the burden of satisfying the court?

With service ex juris, usually the plaintiffRest of the time, usually the defendant

Unofficially, what courts are really concerned about is forum shopping: is BC chosen because its truly the proper place to hear the case, or because there is some particular advantage to the plaintiff to bring the case in BC?

Has the forum been selected for bona fide reasons?However, conflicts is mostly about litigation strategy the choice of forum is very much a strategic consideration.

Page 29: INTRODUCTION - University of Victoria - Web.UVic.caweb.uvic.ca/~lssweb/wp-content/outline_uploads/76... · Web viewDow Corning (2000) (BCCA) Facts: Class actions for negligence in

Discretion as to the exercise of territorial competence – CJA s. 11CJA s. 11(1): After considering the interests of the parties to a proceeding and the ends of justice, a court may decline to exercise its territorial competence in the proceeding on the ground that a court of another state is a more appropriate forum in which to hear the proceeding.

(2) A court, in deciding the question of whether it or a court outside British Columbia is the more appropriate forum in which to hear a proceeding, must consider the circumstances relevant to the proceeding, including

(a) the comparative convenience and expense for the parties to the proceeding and for their witnesses, in litigating in the court or in any alternative forum, (b) the law to be applied to issues in the proceeding, (c) the desirability of avoiding multiplicity of legal proceedings, (d) the desirability of avoiding conflicting decisions in different courts, (e) the enforcement of an eventual judgment, and (f) the fair and efficient working of the Canadian legal system as a whole.

Teck Cominco (BCCA)Under CJA s. 11, court required to consider if a court in another jurisdiction is a more appropriate, or the most appropriate, forumAct seems to codify rather than effect a substantial change to the previous law.

Court points out that the list of factors is not exhaustive – factors are phrased differently, but they are the same principles that have been applied for years.

CJA s. 11(2) does not change the weighing of the factors – remains discretionary dependant on the facts of each case.

Development of Forum Non ConveniensHistorical Approach

English law in this area was largely adopted by the Canadian courts. Old English rule: an English court would be very reluctant to decline jurisdiction if the defendant had been served in the jurisdiction. Defendant had to show that:

Holding the proceedings in England would be oppressive, vexatious or an abuse of process and,That a stay of proceedings would not be unjust to the plaintiff.

Difficult test to meet – was rare to stay English proceedings Consistent with view that the English court system was inherently superior

The Atlantic Star (1974): landmark decision that non-English legal systems worthy of greater respect and deference than courts had been showing discretion to stay English proceedings in favour of foreign proceedings should be exercised more frequently

Lord Denning: “You may call this forum shopping, but if it is then it’s a good place to shop!

Page 30: INTRODUCTION - University of Victoria - Web.UVic.caweb.uvic.ca/~lssweb/wp-content/outline_uploads/76... · Web viewDow Corning (2000) (BCCA) Facts: Class actions for negligence in

MacShannon (1978) – opened up forum non-conveniens in EnglandIn order to justify a stay, two conditions:

Defendant must satisfy the court that there is another jurisdiction where the case can be heard properly and at less expenseMust not deprive the plaintiff of juridical advantage.

Characteristics of the Test

Test: Is there a clearly more appropriate forum? Which has the most real and substantial connection? What was the natural forum? What was in the expectations of the parties?

TEST: what is clearly the more appropriate forum?historically: (1) more convenient forum (2) deprived of juridical advantage (McShannon)now, juridical advantage is just another factor (NOT a separate consideration that has to be met) – Spilidia approach adopted in Amchem burden of proof: services in juris def must clearly establish that a more appropriate forum exists somewhere else. If able to establish that there is another court that is pf the appropriate forum then burden shifts to plf to show justice requires that the trial should take place in the original country. Deference to the plf knocked out where there is forum shopping or fleeting presence (can be def fleeting presence) service ex juris plf has some burden to show that forum chosen is appropriate. Same burden on def, but will be easier to show. Comity will be a bigger factor.Factors to Consider – compiled by Justice Low (BC SC)

where each party resideswhere each party carries on businesswhere the cause of action arosewhere the loss or damage occurredany juridical advantage to the plf in this particular jurisdiction, to the defpotential witnesses – convenience or inconveniencecosts of conducting the litigation, cost to the litigants, perhaps court costs as well whose law is going to be applied – applicable substantive law difficulty/cost of proving foreign law parallel proceedings forum shopping service ex juris or in juris

Juridical advantage includes consideration such as:procedural differences such as discovery, types of damages, limitation rules, rules of evidence,power to award interest

Page 31: INTRODUCTION - University of Victoria - Web.UVic.caweb.uvic.ca/~lssweb/wp-content/outline_uploads/76... · Web viewDow Corning (2000) (BCCA) Facts: Class actions for negligence in

Spiliada Maritime v. Cansulex (1987) (HOL)Facts: Ship is registered in Liberia. Loaded with wet sulphur in Canada. Damage was incurred. Plaintiff, a Liberian company (ship owners) sued the Canadian company in an English court. (All of the insurers were in London). Leave to serve ex juris was granted by the British court.

Arguments:Defendant challenged jurisdiction: argued that the BC court was more appropriate since that is where the damage took place and where the defendants (and their assets) were.Litigation strategy: plaintiff was suing in England because the case would have been time barred under BC law.

Reasoning: A judge may decline to take a case where there is another jurisdiction that is more suitable for the parties.

However, the burden is on the claimant to establish that the foreign forum is clearly or distinctly better.

Term forum non conveniens is not about convenience. Rather, what is at stake is suitability or appropriateness of the forum “the natural forum” is one where the parties would reasonably expect the case to be heard. While the P has a degree of choice in forum, this will not overwhelm the court.

Many factors can be considered to determine the "natural forum" including:

the availability of witnesses, the applicable law of the matter, the parties' residence or place of business, and other cases related to the subject matter,location of insurers, the possibility for the plaintiff to obtain justice in the foreign jurisdiction. Juridical advantage is not a separate consideration

If a party makes out a claim for a natural forum the opposing side may rebut the claim by showing that justice requires the matter to be heard in the domestic court, otherwise justice would not be done.

General preference for the plaintiff’s choice of forum places a significant burden on the defendant to show that another forum is more appropriate.Distinction between service ex juris and in the jurisdiction goes to the burden:

Service in juris: more difficult to show that there is a more appropriate forum. Service ex juris: easier to show that there may be a more appropriate forum elsewhere.

Underlying principle of fairness and comity requires that regard must be had to all of the parties and the ends of justice, and may lead to a different conclusion in other cases.

Courts don’t want to be used for abuse of process (forum shopping)

Page 32: INTRODUCTION - University of Victoria - Web.UVic.caweb.uvic.ca/~lssweb/wp-content/outline_uploads/76... · Web viewDow Corning (2000) (BCCA) Facts: Class actions for negligence in

In this case:If the only reason they chose England was because of the limitation period, that’s not a good enough reason. If the plaintiff is simply forum shopping, the court will not recognize the claim. But if the plaintiff acted reasonably and there was a reasonable connection with the forum, then at least regarding limitation periods, court will look beyond that.

Applying the facts:Court notes that the location of the witnesses was in EnglandThere is another related case going on in England between the renter and the ship owner. The contract of rental generally says that the law of England will apply. Essentially the same set of facts – Crucial pointCommon insurers: most of the insurance in admiralty is done in England.All the work and all the learning has taken place in England based on previous counsel.Loading did take place in BCHeld:

Held: BC is not a more appropriate forum than England. The expenses alone and the existence of a B.C. limitation period was enough to allow the claim to come to England.

Note: Even if the court had granted a stay and the case went to BC, court would have conditioned the stay on the parties waiving their limitation bar (lose that juridical advantage).

472900 BC Ltd. v. Thrifty Canada (1998) (BCCA) Facts: Thrifty Car rental, an Ontario-based company, had a contract with a franchise outlet at Vancouver Airport. Parallel proceeding:

Thrifty suing 472 in Ontario for non-payment472 suing Thrifty for inducement to enter contract in BC

Contract had a clause within it that said that the parties atorne to the jurisdiction of the Ontario courts Ontario court by contract has jurisdiction simpliciter.

Ontario Case472 applied to the Ontario court to have them decline jurisdiction.Ontario court did believe themselves to be an appropriate court, or at least, that they hadn’t made out the case that BC would be a more appropriate court.

472 also sued Thrifty in BC for inducement to enter a contract:After winning in Ontario, Thrifty tried to argue that although BC had jurisdiction simpliciter (not in dispute), asked them to stay the proceedings in BC.Chambers judge declined Thrifty’s application did not make out case that Ontario would be more appropriate forum.Thrifty appealed

Reasoning: Forum non conveniens now based primarily on the concept of comity, particularly where there is the possibility of concurrent proceedings.

The danger of conflicting decisions if two actions were to proceed concurrently in two jurisdictions is a significant one

Page 33: INTRODUCTION - University of Victoria - Web.UVic.caweb.uvic.ca/~lssweb/wp-content/outline_uploads/76... · Web viewDow Corning (2000) (BCCA) Facts: Class actions for negligence in

“Comity demands that such a situation should not be permitted to occur as between courts of two civilised and friendly states. It is a recipe for confusion and injustice.”

Comity, which played no part in the old rule, is now a major consideration. Parallel actions dealing with the same subject matter must now be avoided unless the party resisting the application to stay can demonstrate possible loss of a juridical advantage. Right of the plaintiff to sue in the court of choice is now not as significant of a factor.Primary purpose of forum non conveniens is to avoid multiple proceedings and conflicting judgements.

Although plaintiff choice of forum used to be paramount in old English cases, in the Canadian context, the court notes that in Morgaurd a higher level of comity is to be expected among sister provinces in a federal state (less so for international cases)

High threshold to get a stay of proceedings

Juridical advantage is a factor to be weighed:Plaintiff may show “cogent evidence which objectively demonstrates that that there is some personal or judicial advantage that would be available to him only in the [present forum] that is of such importance that it would cause injustice to him to deprive him of it.”There must be an injustice to move the case this may work to allow you to keep the action in the forum (high threshold)

Held: Allowed the stay of proceedings case to be moved to OntarioPersuaded by the comity between the provinces472 had already gone to Ontario to try and invoke forum non conveniens and had lostProceedings in BC and those in Ontario arose out of the very same issues (really a threshold question) contract was the basis of the dispute

Ontario court itself had already dealt with the issue of forum non conveniensAllowing the action in BC would be to allow concurrent proceedings on essentially the same issue.

Comment: Case is important because shows that parallel proceedings can influence where the case should be heard.

Westec Aerospace v. Raytheon (1999) (BCCA)Facts: Contract between Westec, a BC company, a Raytheon, a Kansas company, involving the transfer of computer technology from BC to Kansas. Contract ends. WestTech argues that Raytheon has not returned source codes argue this amounts to breach of contract in BC.

Raytheon went to a Kansas court to get a declaration that they had not breached the contract.Westec commences a breach of contract case in BC. Raytheon comes to BC and argues that Kansas is the appropriate forum to hear the case and that they already have a case commenced in Kansas.

Page 34: INTRODUCTION - University of Victoria - Web.UVic.caweb.uvic.ca/~lssweb/wp-content/outline_uploads/76... · Web viewDow Corning (2000) (BCCA) Facts: Class actions for negligence in

Concern of the court is the ongoing problem of the race to the court house:

Comity, if applied with vigour, would basically result in that the later court always defers to the earlier court (assuming existence of JS)Unavoidable that such a race will occur: attempting to minimize inefficiency to some extent will inevitably favour the court where the action first originated.BUT: Will only defer to the first court where the second court believes that the initial court house is an appropriate forumIf you had a rule which permitted parallel proceedings, then there might not be a race to file, but rather a race to judgement advantage goes to the first party to obtain judgement

Held: In this set of facts, held that both Kansas and BC were appropriate forums. Where both forums are appropriate, the race to file will be very influential. Comity suggests that the race to file will perhaps be a determining fact.

Here, Kansas was an appropriate forum BC conceded jurisdictionSufficient connection that there was a reasonably expectation that the case could be tried in Kansas. Comity played a role, even though an international case

Sydney Steel v. CNR (1998) (NSCA)Facts: Plaintiff sold steel rails to the defendant under a contract, to be governed by the law of Quebec.

Plaintiff brings action for non-payment in Nova Scotia. Defendant brings its own action against Sydney steal in Quebec for breach of contract.

Sydney Steel goes to Quebec and requests a stay of proceedings since action already being brought in Nova Scotia.

Quebec court denies stay Quebec was “not an inappropriate forum” and the Quebec law applied (as in contract).

CNR comes to Nova Scotia and requests a stay of the Nova Scotia action. Nova Scotia denies the stay CNR could not show that Quebec was the more appropriate forum. Nova Scotia has broad discovery rules not available in Quebec = could be a loss of juridical advantage to Sydney Steel.

Therefore, action proceeds in both jurisdictions!Some argument that the litigation may have involved different issues not true parallel proceedings.

Reasoning: Quebec did not say that they were clearly the more appropriate forum, but rather that they were essentially equal on a balance of convenience.

If comity was to be applied, then suggests that action should be held in the forum where the action was commenced first.

Page 35: INTRODUCTION - University of Victoria - Web.UVic.caweb.uvic.ca/~lssweb/wp-content/outline_uploads/76... · Web viewDow Corning (2000) (BCCA) Facts: Class actions for negligence in

In some ways, NS just annoyed that Quebec didn’t recognize that they had jurisdiction first.

Teck Cominco v. Lloyds / Lombard (2007) (BCCA)Leading case on parallel proceedings in the international context

Facts: Action between Teck Cominco, a BC mining company and Lloyds and Lombard, two insurance companies. Insurance companies argue that they are not responsible to Teck Cominco for action or damages sought by third parties for environmental damages.Four potential sites of action in BC: Vancouver Island, Port McNeill, Pinchy Lake and Trail.

In Trail, BC, there had been discharges of crud from the mining operation into the Columbia river. All the crud was released under permit from the government. Columbia River flows into Washington state. Crud allegedly caused damages in Washington State. Existing lawsuit commenced in Washington based on this damage. Teck Cominco argues that the US court does not have jurisdiction over the matter.

BC ActionIn BC, the insurance companies are bringing a case to get it declared that they had no obligations to defend or indemnify Teck Cominco insurance policies are not at stake.

Washington ActionIn Washington, Teck Cominco goes to court and argues that by declaration, the insurance companies are required to defend and indemnify them. Tech Cominco requests an order from the court to prohibit the insurance companies from proceeding in another court anti-suit injunctionThe insurance companies want a dismissal, argued both on JS and more importantly on FNC.

Back to BC:Lloyd’s finds out about the anti-suit injunction in Washington and goes to BC to get an order to prevent Teck Cominco from prohibiting the action in Washington – essentially an anti- anti-suit injunction!

Race to the court house:Tech Cominco wins by filing at 12:01AM!Teck Cominco argues that because they won the race to the court house and the race to the judgement, that is virtually conclusive Washington court wins in terms of parallel / similar proceedings and BC should not exercise jurisdiction

Race to judgement:Washington court issues judgment first: Washington court is an appropriate forumBC court issues judgement a few days later: BC is also an appropriate forum!

Therefore, both cases proceed! Case is tried in BC generally ignore the action down in Washington

Page 36: INTRODUCTION - University of Victoria - Web.UVic.caweb.uvic.ca/~lssweb/wp-content/outline_uploads/76... · Web viewDow Corning (2000) (BCCA) Facts: Class actions for negligence in

Washington court did recognize that BC likely had the stronger connection, but felt compelled to take the case for public interest reasons no one in Canada was really interested in the case and the environmental damage was occurring in Washington.

Chambers Judge:Lloyd’s had the contract with Teck Cominco in Canadian dollars through a Canadian broker

Very little connection with WashingtonNo choice of forum or choice of law clause in the contract

Lombard also had contract with Canadian basis No connection to WashingtonNo choice of forum or choice of law clause

Damages did occur in WashingtonTeck Cominco does have subsidiaries in the US, but they were not involved in polluting the river therefore, no real connection between Teck Cominco and Washington.

Held: BC is the more appropriate forum will exercise jurisdiction

BCCA decision:Discusses CJA s. 11 and its relationship to previous cases What the act does is codify, but all of the old cases remain valid and important in understanding s. 11.

Issue: The relationship between the CJA and the existing case law – case law is still valid.

Application of CJA s. 11(2):(a) the comparative convenience and expense for the parties to the proceeding and for their witnesses, in litigating in the court or in any alternative forum,

There are additional claims beyond just the state of Washington all of this point to BC

(b) the law to be applied to issues in the proceeding, “The parties would reasonably have expected that since the policies cover global risks, they would be interpreted according to British Columbia law, rather than the law of the site of the particular damage in question in each case.”

(c) the desirability of avoiding multiplicity of legal proceedings, (d) the desirability of avoiding conflicting decisions in different courts, (e) the enforcement of an eventual judgment, and

Not an issue(f) the fair and efficient working of the Canadian legal system as a whole.

Doesn’t matter so much in an international case

To restate briefly the connections in this case:Teck Cominco is a Canadian corporation having its head office in BCthe site of the smelter (and thus the conduct giving rise to the action) is in BC the Insurers have obvious connections with BC

Page 37: INTRODUCTION - University of Victoria - Web.UVic.caweb.uvic.ca/~lssweb/wp-content/outline_uploads/76... · Web viewDow Corning (2000) (BCCA) Facts: Class actions for negligence in

the insurance policies were purchased largely in British Columbia or through BC brokersand the B.C. coverage actions include three claims or potential claims that Teck Cominco has not sued on, and could not sue on, in Washington State.

Held: There was ample evidence to support the Chambers judge's conclusion that British Columbia was the more appropriate forum and that the connections of the parties and the litigation to Washington were weak in comparison.

Forum shopping:Teck Cominco argued that the chambers judge had tarnished their name and biased the decision by suggesting that they were forum shopping

However, court tend to avoid the use of the term “forum shopping”By commencing its proceedings in Washington State, Teck Cominco was hoping to avail itself of the advantages of Washington law for insured partiesIt is not that parties are not entitled to pursue their own self-interest when it comes to choice of forum, but that they are not entitled to invoke the laws of a jurisdiction with which they have little or no connection, in order to avoid the laws of the jurisdiction which is the most appropriate forum for trying the dispute.

Parallel proceedings:At the very least, complex analysis precludes a simplistic approach that defers to the first court to assert jurisdiction. Such an approach would be very close to, and perhaps equally objectionable as, the old rule of deference to the plaintiff's choice

Parallel proceedings are important, but only a factor to be weighed not determinativeMore likely to be the determinative value in a Canadian case due to greater comity

Race to the court house in not the end of the issueWashington court is not more appropriate this is the test even in a parallel proceedings context.

Reviews of cases in Tech Cominco:

Distinguishable from Thrifty: Ontario choice of law clause and Ontario choice of forum clause definitively tied to OntarioLots of connections to OntarioComity within the federal context (sister provinces)

Distinguishable from Westech:Court held that both jurisdictions were appropriate connections to both forumsNo loss of juridical advantage by granting the stayWhere even balance, parallel proceedings more determinative

Analagous to Western Union Case:

Page 38: INTRODUCTION - University of Victoria - Web.UVic.caweb.uvic.ca/~lssweb/wp-content/outline_uploads/76... · Web viewDow Corning (2000) (BCCA) Facts: Class actions for negligence in

Facts (very similar to Teck Cominco): Insurance companies not wanting to provide coverage to BC company in California. BC insurance company. BC company sells product into California, though they are not authorized to conduct business in California. Class action suit brought against BC company in California. BC company wants the insurance to apply in California. In BC, the insurance company argues that they don’t cover whatever is happening in California.

California goes ahead with case.BC court also goes ahead.

Held: No new test to apply here just because there are parallel proceedings doesn’t change the FNC test.

The fact that California went first shouldn’t change the test in BC. Merely a factor still have to determine which of the courts has the closest real and substantial connection

Harrison v. Hederley Transport (BCSC)Facts: Plaintiff is hurt in a car accident in BC as a passenger in a car insured in Alberta. Harrison commences an action both in BC and Alberta (brought in Alberta to avoid limitation periods).

Argument: Alberta truck company comes to BC to say that they should not exercise jurisdiction (prefer case in Alberta)

Held: BC not the more appropriate forum and stay is granted. Law to be applied is BC: but proving BC law in Alberta court not that difficult – not a huge issueBig issue is avoiding multiplicity: everybody else involved in the lawsuit is suing in Alberta (5 actions in Alberta) even the plaintiff has commenced action in Alberta!

Parallel proceedings is just another factor:Seems to have more influence if you are in fact first to the court house and parallel proceedings are both in CanadaComity has a greater impact when there are actions coming out of the same issuesAll of the factors in FNC apply in the case of parallel proceedingsFNC always in the discretion of the court

Court Jurisdiction Selection ClausesContracts will often specify where court action must occur or may specify that disputes must be dealt with through arbitration.Courts are becoming increasingly deferential to these types of clauses, but this doesn’t mean the court must adhere to them

Recognize the primacy of both choice of location and choice of arbitration

Court jurisdiction selection clauses:There is a difference between a choice of forum clause and a choice of law clause:

Page 39: INTRODUCTION - University of Victoria - Web.UVic.caweb.uvic.ca/~lssweb/wp-content/outline_uploads/76... · Web viewDow Corning (2000) (BCCA) Facts: Class actions for negligence in

Just because the contract says that the law of certain forum applies, doesn’t mean that another forum won’t have jurisdiction can apply foreign law in a local forum

Care must be taken with the choice of forum clause to avoid being poorly drafted

Need to clearly specify subject matter of the disputeE.g. “all disputes arising out of the contract” might not necessarily cover a tort

Just because you specify one jurisdiction doesn’t necessarily exclude all others Should be clear in stating that the clause is exclusive: “has exclusive jurisdiction”Under the BC CJA, jurisdiction simpliciter can be given to BC under contract without there being any real or substantial connection to BC presumption that the contract itself is enough to atorne to the BC jurisdiction.

CJA s. 10(e)(ii): Real and substantial connection is presumed where the contract specifies BC law as its choice of law

Outlen Agencies v. Knolloffice Inc. (1988) (PEI CA)Facts: Parties entered in to a sales representation arrangement. PEI company was to sell goods of the Quebec defendant company in the Maritime region. Term of contract was to be 1 year, but agreement continued without renewing the contract. A dispute developed: PEI company wants to sue in PEI, but Quebec company want the case to be heard in Quebec - relies on the contract:

choice of forum clause any legal action is to be heard in Quebecand choice of law clause according to the laws of Quebec.

Issue: Whether the written contract continued to govern the agreement after its expiry?

The courts would uphold a properly framed contractual choice of forum unless the balance of convenience massively favoured an opposite conclusion.

Much heavier burden on someone attacking a choice of forum clause than under FNCHowever, Court does have authority to override choice of forum clauses

Held: Concludes that there is no massive balance in favour of PEI so as to displace the Quebec choice of forum clause case must be moved to Quebec

References the leading Enlgish authority: The Eleftheria (1969)Where plaintiffs sue in England in breach of an agreement to refer disputes to a foreign court, and the defendants apply for a stay, the English court, assuming the claim to be otherwise within its jurisdiction, is not bound to grant a stay but has a discretion whether to do so or not. The discretion should be exercised by granting a stay unless strong cause for not doing so is shown.

The burden of proving such strong cause is on the challenger (party seeking overturn the choice of forum clause)

Page 40: INTRODUCTION - University of Victoria - Web.UVic.caweb.uvic.ca/~lssweb/wp-content/outline_uploads/76... · Web viewDow Corning (2000) (BCCA) Facts: Class actions for negligence in

In exercising its discretion, the court should take into account all the circumstances of the particular case tend to focus on the expectations of the partiesIn particular, but without prejudice to, the following matters, where they arise may properly be regarded (note: these considerations are very much like those used under FNC): (a) In what country the evidence on the issues of fact is situated, or more readily available, and the effect of that on the relative convenience and expense of trial as between the English and foreign courts; (b) Whether the law of the foreign court applies and, if so, whether it differs from, English law in any material respects; (c) With what country either party is connected, and how closely; (d) Whether the defendants genuinely desire trial in the foreign country, or are only seeking procedural advantages (i.e. forum shopping); (e) Whether the plaintiffs would be prejudiced by having to sue in the foreign court because they would

(i) be deprived of security for that claim, (ii) be unable to enforce any judgment obtained, (iii) be faced with a time bar not applicable in England, or (iv) for political, racial, religious or other reasons be unlikely to get a fair trial.

Pompey Industrie v. ECU-Line (2003) (SCC)Facts: Contract for sale of a photo processor to be shipped by ship and rail. Seller in France. Buyer in Seattle. Shipped from France to Antwerp, Belgium shipped to Montreal rail from Montreal to Seattle.

Forum selection clause: bill of lading is “governed by the law of Belgium and any claim or dispute arising hereunder or in connection herewith shall be determined by the courts in Antwerp and no other courts.”Eculine, the French shipping company, wants to bring action in Belgium. Pompey, the buyer, want to bring action in Montreal (for some reason?)

Reasoning: “Forum selection clauses are common components of international commercial transactions, and are particularly common in bills of lading.  They have, in short, “been applied for ages in the industry and by the courts”

These clauses are generally to be encouraged by the courts as they create certainty and security in transaction, derivatives of order and fairness, which are critical components of private international law - Morguard

“In the context of international commerce, order and fairness have been achieved at least in part by application of the “strong cause” test.”  This test rightly imposes the burden on the plaintiff to satisfy the court that there is good reason it should not be bound by the forum selection clause. 

Qualification: the forum selection clause itself must have some semblance of rationality as to why that court was chosen.

In any event, the “strong cause” test provides sufficient leeway for judges to take improper motives into consideration in relevant cases and prevent defendants from relying on forum selection clauses to gain an unfair procedural advantage.

Page 41: INTRODUCTION - University of Victoria - Web.UVic.caweb.uvic.ca/~lssweb/wp-content/outline_uploads/76... · Web viewDow Corning (2000) (BCCA) Facts: Class actions for negligence in

Distinction between test for forum non conveniens and forum selection clause

“Strong cause” test from Eleftheria remains relevant gives full weight to what the parties intended in the contract.Pompey argued that forum selection clause should be treated that same as FNC

Court rejects this argument: because the forum selection clause is in the contract, a different test has to be applied – “strong cause test”While there is a similarity to the factors considered under forum non conveniens, there’s a much higher burden to overcome a forum selection clause

Applying the Facts:ECU-Line prefers to litigate in a familiar jurisdiction (Belgium)Do not go to Belgium to seek a procedural advantage There are reasonable connection to Belgium: forum selection clause, shipped through Belgium, Belgian and French witnessesTime bar has been waved by both partiesEnforcement of the Belgian judgement should not be a problem.

Antwerp court conducts its proceedings in Flemish!Base decision strictly on documents – preclude witnesses and witness cross-examinationMore likely to be delayed in Belgium

Held: Bill of lading is binding on Pompey and the selection clause wins out case to be heard in Belgium

Arbitration ClausesAlthough the results of arbitration clauses and forum selection clauses are largely the same, the procedure for the two is quite different.

For arbitration clauses, there is legislation that changes the processLock out under an arbitration clause is more complete than under a forum selection clause

BC LegislationInternational Commercial Arbitration Act

Adopts international law into BCOnly applies to international disputes explicitly excludes interprovincial disputesS. 8 stipulates that the court in BC is to stay an action where there is an arbitration clause unless the agreement as a whole is null and void, inoperative or incapable of being performed.

Not just the clause, but the entire agreement! Test is that you must meet the section 8 requirements – now very different from FNC

With stays, the discretion of the court is narrowly constrained by the legislation. Court has a lot less control over its own powers.

Commercial Arbitration ActEssentially applies the international act to interprovincial issues

Page 42: INTRODUCTION - University of Victoria - Web.UVic.caweb.uvic.ca/~lssweb/wp-content/outline_uploads/76... · Web viewDow Corning (2000) (BCCA) Facts: Class actions for negligence in

Foreign Arbitral Awards ActLargely the same as the International Commercial Arbitration Act

BWV Investments Ltd. v. Saskferco Products Inc. (1993) (Sask. QB)Facts: German project contractor had a sub-contract with BWV (a Toronto company) to construct a nitrogen fertilizer plant in Saskatchewan for Saskferco. The contract contained an arbitration and choice of law clause: that Swiss law to be applied, arbitration to be in Zurich. BWV claims that they owed money and also claim under the Builders Lien Act. German company argues that action should be stayed, and the matter should be arbitrated in Switzerland (based on the Sask. International Commercial Arbitration Act).

Issue: how does the arbitration legislation interact with the Builders Lien legislation? Does it amount to making the arbitration agreement null and void, inoperative or incapable of being performed.

Clearly a case of conflicting legislation

Reasoning: Court looks at practice in other jurisdictions (England, France, US) and finds increasing deference to commercial arbitration agreements

Starting point: respect freedom of contract as a matter of public policy Give effect to the intentions of the parties Facilitate predictability in dispute resolution, commercial activity internationality Consistency between jurisdictions in dispute resolution Encourage international commercial activity

Found that the parties were bound to defer to the contract, unless the agreement was null and void, inoperative or incapable of being performed.

Reading down the Builders Lien Act:Purpose of the Builders Lien Act (to make sure owner doesn’t receive the benefit without paying) does NOT suggest that this is the only way of determining the quantum of damages where disputes have arisen nothing to stop parties from “opting-out” of the Builders Lein Act

Read the legislation down to give priority to the arbitration agreement in the contract.

Held: Court decided that the arbitration clause should be upheld and the dispute should be arbitrated in Switzerland.

Need to respect freedom of contract entered into by sophisticated parties leads to certainty and encourages international commercial arbitration.

Comment: The Builders Lien Act was actually much more exclusive and strong then the courts read it to be. However, this act shows that an air-tight arbitration clause can still hold supremacy.

Court showing tremendous deference, interpreting domestic legislation even in the face of contrary evidenceHOWEVER, there is still a really high burden to go after an arbitration clause, even where looks that you have conflicting legislation

Page 43: INTRODUCTION - University of Victoria - Web.UVic.caweb.uvic.ca/~lssweb/wp-content/outline_uploads/76... · Web viewDow Corning (2000) (BCCA) Facts: Class actions for negligence in

Anti-Suit InjunctionsDefinition: An injunction issued by a court in jurisdiction A against a party in jurisdiction B seeking to prevent this party from bringing action in jurisdiction B.

If the injunctive party proceeds with an action in jurisdiction B, they are then subject to an order of contempt of court in jurisdiction A and subject to damages in jurisdiction A.

Definition from Aerospatiale:Where the court decides to grant an injunction restraining proceedings in a foreign court, its order is directed not against the foreign court but against the parties so proceeding, or threatening to proceed. If a party who is ordered to discontinue an action sees fit to disobey the order and proceed with the action, the court acts upon the party for their disobedience of this court.If the party against which the injunction is sought has little connection to the jurisdiction, then its unlikely that an injunction will be granted, since it will likely be ineffective.

Even if the injunction is issued, a person with no connection will likely not be affected Unlikely that the courts in jurisdiction B would enforce the injunction, or that they would enforce a damages judgement, from a court in jurisdiction A.

Ordering an anti-suit injunction can put a strain on comity essentially one court trying to control what another court does, by way of the parties.

Injunctions are equitable orders very much at the discretion of the courtRarely granted courts reluctant to grant because of problems with comity.

Societe nationale Industrielle Aerospatiale v Lee Kui Jak (1987) (Privy Council)Facts: Helicopter falls out of the sky in Brunei and 12 people are killed. Helicopter was manufactured by Aerospatiale, a French company. Helicopter was owned by an English company. Brunei government ordered a transportation safety inquiry determined that there was a mechanical malfunction and a failure to advise regarding regular maintenance.

Widow of one of the deceased brings action all over: France, Brunei and Texas.Brought in Texas on the grounds that Aerospatiale did business in Texas.

Texas Wrongful Death statute: allowed the action to be brought in Texas simply on the basis that the cause of death might have involved a company that had a subsidiary in Texas. Suing in Texas because lenient products liability law and high damage awards.

Texas ActionMotion by Areospatiale in Texas to dismiss the case on the grounds of forum non conveniens. Widow counters that the forum non conveniens has no application under the Texas statute.

Also argue that the court should not use discretion

Page 44: INTRODUCTION - University of Victoria - Web.UVic.caweb.uvic.ca/~lssweb/wp-content/outline_uploads/76... · Web viewDow Corning (2000) (BCCA) Facts: Class actions for negligence in

Texas court dismissed the Aerospatiale motion, but gave no reasons (not required to!)

Texas case to proceed - passes forum non conveniens

Brunei ActionIn Brunei, Aerospatiale tries to get an anti-suit injunction to prevent the widow from proceedings in TexasUnsuccessful in getting the injunction. Appeal all the way to the Privy Council in England

Held: Court notes several starting principles:An injunction shall be granted where the ends of justice require itAn anti-suit injunction is not a declaration of war on the other court, but rather is issued against the individuals partyWill only be granted where the defendant is “amenable to the jurisdiction” – i.e. where its going to be effectiveMust be careful: many comity issues at stake – indirectly affect foreign courts

Privy council tries to create two categories of anti-suit injunctions: Where the injunction is designed to protect English court jurisdiction –

possibility that a foreign judgement would compromise English court jurisdiction. I.e. don’t want interference from foreign courts – easier to get an injunction in these circumstances

Where there are (or possibility of) parallel proceedings – purpose of the injunction is to quiet the parallel proceedings

Goes more to court efficiency – more difficult to get the injunctionWill only be granted where the foreign proceedings would be vexatious or oppressive

Of these two categories, Aerospatiale fits into the second category Must show that the action in the Texas court is vexatious or oppressive.

The fact that there are parallel proceedings is not sufficient to be granted an anti-suit injunction – much higher burden

Just because the court is being heard in an inconvenient location is not vexatious or oppressive

However, abuses of courts can occur where parties go to a jurisdiction with little or no connection to the parties simply to get higher damages

Alluding to forum shoppingForum non conveniens is a way to balance jurisdictions if the foreign court doesn’t seem to be applying forum non conveniens properly and is therefore not respecting comity, this may be grounds to grant an anti-suit injunction.

The action of the foreign court is considered to be oppressive

Quote:"Their Lordships refer, in particular, to the fact that litigants may now be encouraged to proceed in foreign jurisdictions, having no connection with the subject matter of the dispute, which exercise an exceptionally broad jurisdiction and which offer great inducements, in particular greatly enhanced, even punitive, damages, that they may tempt litigants to pursue their remedies there.

Page 45: INTRODUCTION - University of Victoria - Web.UVic.caweb.uvic.ca/~lssweb/wp-content/outline_uploads/76... · Web viewDow Corning (2000) (BCCA) Facts: Class actions for negligence in

In normal circumstances, application of the now very widely recognised principle of forum non conveniens should ensure that the foreign court will itself, where appropriate, decline to exercise its own jurisdiction . . . But a stay may not be granted; and if, in particular, the English court concludes that it is the natural forum for the adjudication of the relevant dispute, and that by proceeding in the foreign court the plaintiff is acting oppressively, the English court may, in the interests of justice, grant an injunction restraining the plaintiff from pursuing the proceedings in the foreign court."

Court rejects the argument that the test for forum non conveniens and for anti-suit injunctions are so closely related that the Spalida test should actually apply in the same way for anti-suit injunctionsWhatever the relationship between the two might be, there is going to be a separate test for anti-suit injunctions, largely because of the much higher importance of comity considerations in the context of an anti-suit injunction.

Held: In this case, concludes that it’s an appropriate case to issue an anti-suit injunction

Texas had no significant connectionTexas not respecting comityTexas selected only because of possibility of higher damages (forum shopping)

Summary: The test for a stay of proceedings under forum non conveniens and for an anti-suit injunction are not the same

Generally, although many of the considerations under the two tests are the same, the burden to be granted an anti-suit injunction is much higher

Anti-suit injunctions can only be issued where the proceedings in the foreign jurisdiction were vexatious, oppressive and contrary to the ends of justice

Where a foreign court is clearly not adhering to forum non conveniens and exercising jurisdiction improperly, this raises concerns for comity and may be considered as oppressive and thus grounds to issue an anti-suit injunctionCourt granting the anti-suit injunction must be a natural forum – not sufficient on its own, but makes it easier.

Airbus v. Patel (1999) (HoL)Facts: English residents seeking damages from an Airbus crash which took place in India. Residents commence action in India, and Texas for good measure. Airbus goes to India: seeks and receives an anti-suit injunction from the Indian court to prevent the action in Texas – however, Texas ignores this (India has no control over the parties). Residents abandon the case in India and continue in Texas. Airbus then goes to the English court (where there is currently no action) and seeks another anti-suit injunction against the Texas action.

Although the UK residents could have brought an action in the UK, they chose not to

Page 46: INTRODUCTION - University of Victoria - Web.UVic.caweb.uvic.ca/~lssweb/wp-content/outline_uploads/76... · Web viewDow Corning (2000) (BCCA) Facts: Class actions for negligence in

Issue: Whether the English court would grant an anti-suit injunction in circumstances where there was no relevant connection between the English jurisdiction and the proceedings in question, other than the appellants who were resident in England, were subject to the jurisdiction and could effectively be restrained by an injunction granted by an English court.

Reasoning: before granting injunction, court has to be satisfied that it has sufficient connection:

limits imposed by comity would be inconsistent to grant injunction where they have no interest

Interference is NOT justified solely on the grounds that a third foreign country cannot enforce an anti-suit injunction (in this case, India)

Court really suspicious about using the UK courts to accomplish what the foreign courts cannot accomplish “In a world which consists of independent jurisdictions, interference, even indirect interference, by the courts of one jurisdiction with the exercise of the jurisdiction of a foreign court cannot in my opinion be justified by the fact that a third jurisdiction is affected but is powerless to intervene. The basic principle is that only the courts of an interested jurisdiction can act in the matter; and if they are powerless to do so, that will not of itself be enough to justify the courts of another jurisdiction to act in their place.

Held: No anti-suit injunction granted English courts have no connection with the matters in question – it was a choice of the plaintiffs not to sue in England.

This is really an abuse of the process.

Comment: If there had been a stronger connection with the English court, might have been a different result.

Amchem v. BC Workers Compensation Board (1993) (SCC)Facts: Many plaintiffs (some BC resident, some not) involved in an asbestos case. Defendants are all American companies with no connection to Canada. Plaintiffs seek action in Texas. None of the companies are based in Texas, but many manufacture and have offices there. Under Texas standards, this is a reasonable connection to Texas.

Texas found that they had jurisdiction simpliciter to any company carrying on business in TexasAmchem applied for a stay in Texas under forum non conveniens denied because a Texas stature wiped out forum non conveniensAsbestos companies came to BC and applied for an anti-suit injunction to prevent the action in Texas from continuing.

Side issue: non-BC residents also seek an injunction in Texas to prevent the anti-suit injunction in BC.

Lower Court: In BC, the injunction was granted by the lower court on the condition that they atorne to the jurisdiction of BC didn’t want them to come back later and claim that BC didn’t have jurisdiction

Page 47: INTRODUCTION - University of Victoria - Web.UVic.caweb.uvic.ca/~lssweb/wp-content/outline_uploads/76... · Web viewDow Corning (2000) (BCCA) Facts: Class actions for negligence in

Being wary of forum shopping and courts being abused.

Lower court judge relied on all the English cases: permissible to grant an anti-suit injunction where the plaintiff was acting oppressively in the foreign court.

More annoyed by the fact that the Texas statute had wiped out forum non conveniens.

BCCA:Also allowed the anti-suit injunction the absence of the forum non conveniens in the Texas jurisdiction was a big problem. Weak argument: Granting the anti-suit injunction did not compromise the principle of comity since directed at individuals, not the court.

SCC (Sopinka): Reviews all the previous Canadian casesConcludes that there is no consistent approach in Canadian law to anti-suit injunctions. Only certain thing is that anti-suit injunctions should be granted with great caution (not particularly helpful)!Holds that Aerospatiale should be a the foundation for the Canadian approach:

Due regard must be paid to comityHowever, rejects that the emphasis should be on vexatious or oppressive conduct – differs from British approach. Also unimpressed with Lord Goff’s two category approach

Four principles are set out (although they are not really followed in subsequent cases):

Procedural: domestic court should not entertain the application unless (a) foreign action has been commenced or is pending, and (b) party seeking injunction sought a stay in the foreign court (under forum non conveniens) and been denied. “In order to resort to this special remedy consonant with the principles of comity, it is preferable that the decision of the foreign court not be pre-empted until a proceeding has been launched in that court and the applicant for an injunction in the domestic court has sought from the foreign court a stay or other termination of the foreign proceedings and failed.”Want to wait for the other court to consider forum non conveniens if the case is stayed or dismissed in the other jurisdiction, problem solved!

Effectivity: domestic court must be a natural forum, that is, it must have jurisdiction simpliciter real and substantial connection

“…the domestic court must proceed to entertain the application for an injunction but only if it is alleged to be the most appropriate forum and is potentially an appropriate forum.” This will automatically be met if there is already action underway on the same matter in BCMust also take into account that there is an action proceeding in another court and has been accepted there

Comity: consider whether foreign court assumed jurisdiction on a basis that is inconsistent with principles relating to forum non conveniens (as understood by the domestic court).

Page 48: INTRODUCTION - University of Victoria - Web.UVic.caweb.uvic.ca/~lssweb/wp-content/outline_uploads/76... · Web viewDow Corning (2000) (BCCA) Facts: Class actions for negligence in

Anti-suit injunction will only be granted where the foreign court exercised jurisdiction in a manner that was inconsistent with our principles of forum non conveniens Foreign court doesn’t have to use the specific doctrine, as long as they consider the principles. “The foreign court, not having, itself, observed the rules of comity, cannot expect its decision to be respected on the basis of comity.”

Juridical disadvantage tied with unjust result : general rule from Aerospatiale - will not grant an injunction if by doing so they will be depriving the plaintiff of advantages that it would be unjust to deprive them of.

Consider loss of juridical advantage, extent of connection to the forum, loss of personal advantage Trying to avoid forum shopping – need to look at the connections with Texas

If you are legitimately in Texas, it would unjust to deprive you of a juridical advantage in Texas.However, if chose Texas only because of higher possible damages and there was no real connection otherwise, not unjust to deprive you of this advantage

Excerpt:The result of the application of these principles is that when a foreign court assumes jurisdiction on a basis that generally conforms to our rule of private international law relating to the forum non conveniens, that decision will be respected and a Canadian court will not purport to make the decision for the foreign court. 

The policy of our courts with respect to comity demands no less.  If, however, a foreign court assumes jurisdiction on a basis that is inconsistent with our rules of private international law and an injustice results to a litigant or "would-be" litigant in our courts, then the assumption of jurisdiction is inequitable and the party invoking the foreign jurisdiction can be restrained.  The foreign court, not having, itself, observed the rules of comity, cannot expect its decision to be respected on the basis of comity.

Held: Anti-suit injunction overturned – Texas was an appropriate forum. Amchem failed to show a legitimate juridical advantage that it would be unjust to deprive them of

They were in fact carrying on business in Texas an should reasonably have contemplated that they could be sued on Texas

Texas does not have forum non conveniens as we understand it, but does have some inkling of the process

14th amendment – due process: provides a safeguard on states from taking jurisdiction where they have no connectionTexas cannot just take jurisdiction any time it wants – must accord with due process under US constitutional law

As long something like forum non conveniens was applied (in this case, due process), that was sufficient

Page 49: INTRODUCTION - University of Victoria - Web.UVic.caweb.uvic.ca/~lssweb/wp-content/outline_uploads/76... · Web viewDow Corning (2000) (BCCA) Facts: Class actions for negligence in

Hudon v. Geos Language Corporation (1997) (Ont. Div. Crt.)Facts: Houdon employed by Geos to teach English in Japan. Took a vacation in China and was injured and severely disabled. Brought an action in Ontario against the Japanese company claiming that the defendant had promised to provide medical insurance coverage, which it apparently had not done.

Japanese company applies for a stay in Ontario on the grounds of forum non conveniens – dismissed. Japanese defendants did not appeal the dismissal, but did commence proceedings in Japan: sought a declaration that under Japanese law, which expressly governed the contract with the plaintiff, that the Japanese company was not liable in any way for failing to provide holiday medical insurance. Plaintiff applies in Ontario for an anti-suit injunction against the Japanese company from proceeding in Japan

Held: the anti-suit injunction should be granted. The court applied the principles from Amchem:

Procedural: not metPreferable for the person seeking an anti-suit injunction to ask the foreign courts for a stay first. This had not happened here, but that it is not a strict requirement to obtaining an anti-suit injunction (strongly preferable but not an absolute requirement)

Effectivity: Ontario court decided that it was an appropriate forumEvidence relating to the assessment of damages can only be given by Ontario witnesses Japanese witnesses can come if required, but their evidence will not be as essential or controversial as that of the Ontario witnesses. Important, but not conclusive that the Ontario court had already said this is an appropriate forum

Comity: not met:Japanese action was purely declaratory – did not engage in any forum non conveniens type analysis, so no comparison could be made

Injustice in depriving juridical advantage: No injustice to the Japanese defendant to hear the case in Ontario

Geos will be able to prove the relevant law of Japan if the case is tried in OntarioGeos failed to show that it will suffer any legitimate advantagePlaintiff’s physical disabilities mean she would suffer large personal disadvantage by having to travel to Japan for trial.

Comment: Note that only two of the condition from Amchem were met:

Ontario was an appropriate forumNo injustice to the Japanese company in hearing the case in Ontario

Indicates that the Amchem principles, although carefully laid out by Sopinka, are actually fluid. Ted says this is not an abuse of the process, just not strict application result is not unfair.

Page 50: INTRODUCTION - University of Victoria - Web.UVic.caweb.uvic.ca/~lssweb/wp-content/outline_uploads/76... · Web viewDow Corning (2000) (BCCA) Facts: Class actions for negligence in

Gentra Canada v. Lehndorff (1995) (AB CA)Facts: A subsidiary of Lehndorrf borrows money from Gentra. Subsidiary could not repay the money. Lehndorff creates a debt agreement with Gentra to purchase the debt. If Lehndorff does not purchase the debt, then Gentra will be able to realize their security on a mall in Alberta (reposess it). All of these agreements are negotiated in Ontario, with the exception of the mall. Gentra decided they didn’t want the agreement and would rather foreclose on the assets.

Lehndorff commences action in Ontario for breach of contractLehndorff gets a regular injunction in Alberta to prevent seizure of the mall pending the outcome of the caseGentra brings action in Alberta for damage and get possession of the mall

Lehndorff files a defence and files a counterclaim for damages against Gentra

Gentra goes to court in Ontario and requests a stay of proceedings on the ground of forum non conveniens stay is dismissed and action in Ontario proceeds

Reason: failed to show that Alberta was the more appropriate forumHowever, Ontario court can’t deal with land in Alberta!

Issue: did the Ontario court use an inappropriate forum non conveniens test?

Reasoning: Anti-suit injunction granted Ontario should not have taken jurisdiction under forum non conveniens

Failed to consider crucial aspect: Lehndorff incorporated the Ontario lawsuit into the Alberta foreclosure action by pleading it in its Defence and Counterclaim filed in the Alberta action and thereby attorned to the Alberta jurisdiction Alberta is the most and the ONLY appropriate forumOntario court decision was “unreasonable”:

There is no injustice to the Lehnforff in proceeding thereThere is injustice to the Gentra in forcing it to proceed with the same action in two jurisdictions or, at least, in forcing it, in effect, to stay its foreclosure action while the Ontario action proceedsNote: Dissent would have allowed the appeal – give greater importance to comity (doesn’t think that the Ontario decision is so inadequate as to be unreasonable)

Comment: this is illustrative of how difficult it is to resolve problems of concurrent jurisdiction with the current tools of broad discretion.

Left with two actions proceeding seems to promote defensive use of anti-suit injunction (race to the finish), incompatible with comity

Choice of LawChoice of law is part of the common law of British Columbia i.e. the principles that we use in choice of law were developed through the common lawAfter taking jurisdiction, there may be a question about which law appliesDomestic court may apply foreign law in some contextsChoice of law is about deciding what the appropriate lex causi is, the law of the court that will be applied

Page 51: INTRODUCTION - University of Victoria - Web.UVic.caweb.uvic.ca/~lssweb/wp-content/outline_uploads/76... · Web viewDow Corning (2000) (BCCA) Facts: Class actions for negligence in

Choices are:lex fori – law of the domestic forumlex loci delicti – place of the wrong or infringementlex situs – law of the situs or location of the subject matter (location of the land or movable)

Procedural law is always lex foriSometimes difficult distinction between substance and procedure

Foreign law is a fact that must be pleaded by the parties and proven by expert witnesses

No obligation on the other side to do thisTechnically it applies between provinces, but because of the unifying feature of the SCC, courts do look to other provincial laws and interpret them

Choice of law influenced by a number of factors:May be in the contractComityGovernment interest analysisVested rights analysisExpectations of the parties

Three questions arise:When is foreign law to be applied?Which foreign law is to be applied?What is the foreign law that needs to be applied?

When is a foreign law to be applied? Choice of law only becomes a possibility/issue in the BC court if it is plead you must plead that an issue before the BC court should be resolved through reference to foreign law. Must be able to show as a matter of fact that the foreign law will actually lead to a different result than the BC law

BC courts prefers to apply BC substantive lawIn some areas, choice of law does not arise because of international treaties or because the substantive law is essentially the same regardless of where you tend to be.

Even if foreign law may apply, may be a choice not to plead choice of law as it increases complexity and expense

Which foreign law applies?The Anglo-Canadian perspective is fundamentally different from the American perspective in most statesIn Canada (and most of the commonwealth) there is a very mechanical process for determining which law will apply

Three step process:Step 1: Characterization – which juridical category does the case fit within?

Determined by the lex fori (BC court) – whether the subject matter is tort, contract, marriage, etc.

Page 52: INTRODUCTION - University of Victoria - Web.UVic.caweb.uvic.ca/~lssweb/wp-content/outline_uploads/76... · Web viewDow Corning (2000) (BCCA) Facts: Class actions for negligence in

Step 2: What is the connecting factor? For each juridical category, there will be a connecting factor (e.g. tort = lex loci delecti – where the tort occurs, property = lex situs – where the property is). It is the connecting factor that points to the external jurisdiction.

Step 3: Lex causiThe law of the court that is to be applied

Characterization The conflict rules point to the jurisdiction whose laws will govern the subject matter:

Each jurisdiction has its own choice of law regime: Even for those jurisdictions that take the Anglo-Canadian perspective, they may nonetheless have different juridical categories.Even if there is commonality in characterization, there may be variation in the corresponding connecting factors.

E.g. Even where the issue in the case is put in the contractual category, connecting factor could be: Where the contract was formed Where the contract was performed What was in the expectation of the parties

There are many juridical categories, which are determined by the lex fori. May also be subcategories.

Whether a particular set of facts fits within a certain category is determined by the court.

Connecting factor is also determined by the lex fori = may get a very different result in another jurisdictionAllowing the lex fori to make there determinations allows for some degree of predictability, but there may be wide variation in different courts. There may be a fairly high degree of consistency among common law jurisdiction, but varies widely with civil law jurisdictions.However, between the provinces within Canada, there is a fairly high degree of consistency between jurisdictions for both characterization and the corresponding connecting factorsStatutes may affect the juridical category, and more importantly, the applicable connecting factor (particularly true in family law)

REMEMBER: Choice of law is trumped in a number of situations:By legislation – may be explicit or implicit

Particularly prominent in family lawIn contract, by a choice of law clause (except when it isn’t!)

However, clause is subject to being overriddenBy public policy: the BC court will not apply foreign law that goes against the public policy of BC

Usually this is only an argument of last resort – tough to win on this argument!

Page 53: INTRODUCTION - University of Victoria - Web.UVic.caweb.uvic.ca/~lssweb/wp-content/outline_uploads/76... · Web viewDow Corning (2000) (BCCA) Facts: Class actions for negligence in

Choice of law only applies to substantive law, and not to questions of procedure

If the matter is procedural, its up to the lex fori to decide (apply BC procedure)

Choice of law does not apply where what is being brought in is penal law or revenue law

BC courts is not to enforce the penal or revenue laws of a foreign jurisdiction

Problems with the Mechanical/Classical ApproachArbitrariness: use of artificial constructs (such as the locus of a tort), ability to recharacterize an issue to reach a desired result No account taken o f real conflict of interests bw states (not true to assume that the state where the tort takes place necessarily has the greater interest) No systematic account taken of international policies (want efficient ordering of multinational interactions) Alternatives:

Issue particularized rather than categorized breaking down to single choice of law issues (e.g. instead of “tort,” look at standard of care, duty owed, etc.) Proper law (k) approach – look to the real and substantial connection Government interests – look to the conflict of state interests, if a true conflict then chose to apply the forum law The better law – consider what will lead to a substantively just result, looking at the parties expectations, the basic policies underlying the area of law, and the certainty and predictability impact of the result.

US Approach rejected in Tolofson US approach: evaluate substantive law choice based on social policy, justice to litigants

Trade-off between predictability and common-sense Trend that judges will want to apply own law Criticized for being too fluid and flexible

Tolofson v. Jensen (1994) (SCC)Held (La Forest): Decisive and unanimous rejection of the newer tendencies of the American courts, in favour of classical application

“From the general principle that a state has exclusive jurisdiction within its own territories and that other states must under principles of comity respect the exercise of its jurisdiction within its own territory, it seems axiomatic (self-evident) to me that, at least as a general rule, the law to be applied in torts is the law of the place where the activity occurred, i.e., the lex loci delicti.  There are situations, of course, notably where an act occurs in one place but the consequences are directly felt elsewhere, when the issue of where the tort takes place itself raises thorny issues.”“The rule has the advantage of certainty, ease of application and predictability.  Moreover, it would seem to meet normal expectations.  Ordinarily people expect their activities to be governed by the law of the place where they happen to be and expect that concomitant legal benefits and responsibilities will be defined accordingly.  The

Page 54: INTRODUCTION - University of Victoria - Web.UVic.caweb.uvic.ca/~lssweb/wp-content/outline_uploads/76... · Web viewDow Corning (2000) (BCCA) Facts: Class actions for negligence in

government of that place is the only one with power to deal with these activities.”  Also claims easier to apply (Ted says this is nonsense though) May also prevent forum shopping

What is the foreign law?Must prove the foreign law before it can be applied in court!What the foreign law is is a question of fact to be argued in the foreign courtMust be both plead and proven in order to be applied – this is a common law proposition

Under the civil law, if the case appears to require the application of foreign law, then the civil court is required to go out and find out the foreign lawIn the common law system, the party seeking to rely on foreign law must themselves prove it

Amosin v. The Ship “Mercury Bell” (1986) (FCA)Facts: Pilipino sailors working on a Liberian registered vessel. Sailors were hired in Manila and signed individual employment contracts. Once hired on the Mercury Bell, they found out there was a collective agreement on the vessel that allowed for a higher rate of pay. Vessel arrived in Canada, and the sailors decide to sue the ship for the difference in wages.

The Canadian Shipping Act directed the court on the issue of choice of law: provision in the Act stated that any matter relating to a ship or to a person belonging to a ship not otherwise covered under the shipping act is to be covered by the law of the place of the ships registry = Liberian law should apply Because the statute stated the law that should apply, did not need to plead the foreign law Court agreed that is was to be Liberian law that would govern the contract.Unfortunately, no one could prove what the Liberian law was before the court!

Issue: Where it is clear that the foreign law should apply but has not been proven, does the lex fori include the common law or also statutes?

Held: In the absence in the proof of foreign law, the foreign law cannot be applied.

Therefore, only alternative is to apply Canadian law, but uncertain if should apply just common law, or statutes as well.Court noted that scholars appear to distinguish the two situation – some forums applied only the common law, while other applied the statutes (not very helpful!) Court decides that the distinction between common law and statute is artificial Instead, finds it better to differentiate between the general law of the forum and aspects of the law that were intended to have only local application

Page 55: INTRODUCTION - University of Victoria - Web.UVic.caweb.uvic.ca/~lssweb/wp-content/outline_uploads/76... · Web viewDow Corning (2000) (BCCA) Facts: Class actions for negligence in

Result: Canadian labour code should apply, but only partially. Canadian Labour Code should apply to the extent that the collective agreement took precedence over the individual context.

Common practice – similar provisions found in other countriesBUT provisions dealing with the procedures under the Canadian Labour Relations Board not applicable – too local

Comment: Unless the foreign law is really going to help you, not usually worth pleading

May choose not to invoke the foreign law where: (1) it would be pointless to do so (2) where the time and expense required to prove the foreign law would outweigh any advantage to be obtained by its application (3) application of the foreign law would have an unpredictable outcome

How do you prove foreign law?Requires the use of foreign law experts

Foreign lawyersLaw academicsGovernment or business workers

BUT courts not necessarily bound to accept the opinions of the expertsCourts are entitled to draw their own conclusions about the law – fact finding mission

Hunt v. T and N Plc (1993)Facts: Quebec blocking legislation prevented business documents from leaving the province (prevented discovery). Issue: whether the statute is ultra vires or whether it is constitutionally inapplicable to a judicial proceeding in another provinceRule: No reason why judge can’t deal with constitutional issue that incidentally arises in the ordinary course of litigation

Jurisdiction of superior court of a province includes decision as to constitutionalityBUT power limited: must be real and substantial connection to the forum

Held: constitutionality of the law that was to be applied was a question of fact May plead the Quebec law, and then argue that it was not valid on the basis of unconstitutionality follows that the validity of the statute is material Unconstitutional because does not comply with requirement to give full faith and credit to judgements rendered by a court in a sister province inapplicable to proceedings in another province (ultra vires)

Page 56: INTRODUCTION - University of Victoria - Web.UVic.caweb.uvic.ca/~lssweb/wp-content/outline_uploads/76... · Web viewDow Corning (2000) (BCCA) Facts: Class actions for negligence in

Procedure and SubstanceLimitation PeriodsTolofson v. Jensen (1994) (SCC)Facts: Plaintiff injured in car accident where his father was driving. Accident occurred in Saskatchewan. Tolofson and his father were BC residents and the car was registered there. Tolofson brought action in BC against his father and Jensen (a resident in Saskatchewan). Saskatchewan limitation period had expired.

Issue: common law test respecting limitation periods turned on the wording of the statute:

If the statute only barred the remedy – procedural If the substantive right was extinguished – substantiveProblem: artificial distinction, turning on bizarre words trying to determine what was meant; if limitations are procedural will lead to forum shopping

Looks at position historically:CL courts in UK and US says limitation is procedural

This is because of the historical rationale that foreign litigants in the UK ought not to have an advantage that other people don’t haveAlso because common law looked at a substantive right as existing forever, but your ability to utilize the court system to enforce it is limited by statute

Civil law regards it as substantiveCivil law is the opposite, views ability to pursue a remedy as substantiveA limitation destroyed the substantive right

Analysis for choosing between substantive/proceduralRULE: The substantive rights of the parties to an action may be governed by a foreign law, but all maters pertaining to procedure are governed by law of forum

If lex loci delicti will be the substantive law, you should give preference to finding the rule as substantiveIf lex fori will govern, should err toward procedureWhen dealing with internal Canadian issues – limitations periods are substantive

Purpose of the classification is determine which rules will make the machinery of the forum court run smoothly as opposed to those that relate to substantive rightsIf there is doubt as to whether substantive or procedural, then you should err on the side of proceduralThe rule of procedure is about convenience for the forum

Test is : how far can the court go in applying the foreign rules without inconveniencing itself?

Old Reasoning:BC Limitation Act s.9: limitation periods considered to be substantive

Page 57: INTRODUCTION - University of Victoria - Web.UVic.caweb.uvic.ca/~lssweb/wp-content/outline_uploads/76... · Web viewDow Corning (2000) (BCCA) Facts: Class actions for negligence in

Effect of a limitation period is to extinguish the cause of action, not the remedy BUT doesn’t say how BC court is to interpret foreign limitations

s.13: where the foreign law is the lex causi (supposed to apply), and the foreign law is procedural (such that it won’t apply) BC Courts have a choice: can apply the BC limitation OR the foreign limitation, depending on which achieves the most just result – Tolofson nullifies this almost entirely

Held (La Forest): The Saskatchewan limitation rule was properly characterized as substantive and hence P’s action is statute-barred

Comment: Procedure has been changed significantly by Tolofson CL distinction (remedy v. rights) although somewhat discredited by Tolofson, is still applicable and being used in several areas e.g. seizure and sue legislation (debtor-creditor) are substantive even though go to dealing with rights rather than remedies; no fault insurance (considered substantive in BC and when drawing it in from another jurisdiction); maritime liens

Castillo v. Castillo (2005) (SCC)Facts: Car accident in California. Husband and wife are residents of Alberta. Spouses’ insurance companies are suing one another. Action is commenced in Alberta after the California limitation period has expired but before the Alberta limitation period expired.

Issue: Lex loci delecti: law of where the tort occurred applied, in this case California. Therefore, the law of California applies, including the California limitation periodS. 12 of the Alberta Limitation Act – Attempt by the Alberta legislature to have the Alberta limitation period apply for all remedial orders even if the foreign law is to be used.

Looks like an attempt to establish that if you are suing in Alberta, irrespective of what law applies (whether foreign or domestic), that the Alberta limitation period applies.

Held: Alberta courts and Supreme Court will have nothing of this – whatever the Alberta legislature thought they were doing, they can’t do it.

Since the California limitations period applied and had expired prior to the commencement of the action, there was no right of action at the time the appellant initiated her claim in the Alberta court. 

The action is dead because you cannot bring it in California which is where the substantive law is here. Substantial right not to be sued has been acquired in California once limitation has expired, right has been acquired not to be sued.

Section 12 does not purport to revive an action time-barred by the substantive law of the place where the accident occurred.  Had the intention of the legislature been as argued, the legislation would have said so.Legislation is not struck, but merely held to be inoperable in the present circumstances

Page 58: INTRODUCTION - University of Victoria - Web.UVic.caweb.uvic.ca/~lssweb/wp-content/outline_uploads/76... · Web viewDow Corning (2000) (BCCA) Facts: Class actions for negligence in

S. 12 does have an operation, but can only operate if the Alberta limitation period is shorter than the foreign limitation period. The Court’s classification of limitation periods for “choice of law” purposes as substantive rather than procedural did not (and did not purport to) deny the province’s legislative authority over the “Administration of Justice in the Province”.  A foreign jurisdiction, by adopting a limitation period longer than that of Alberta, cannot validly impose on Alberta courts an obligation to hear a case that Alberta, as a matter of its own legislative policy, bars the court from entertaining.

In these circumstances, would still be able to sue in California

Two relevant areas: Seize or sue legislation: legislation that protects the creditor

Must choose whether to sue the debtor or seize the assets – substantive rather than procedural requirement. Reasoning: statutes of this type are convenient to ascertain and apply, which gives them a procedural flavour, but they deal with a substantive requirement.

No fault insurance: restricts civil recourse in tortsHave been viewed as substantive rather than proceduralProviso: not a definitive rule but rather a tendency depends on how the insurance is written:

What is the reach of the insurance legislation? – if it has an extra-territorial effect, then will be held to be substantive.

Legal Status - Substantive v. Procedural DistinctionGeneral Rule: lex fori determines the status BUT, Hamsa demonstrates a peculiar exception but: status in the foreign jurisdiction may be relevant in determining whether the entity can sue

International Assn. of Science and Technology v. Hamza (1995) (AB CA)Facts: matrimonial property action - Swiss society swoops in to say that certain assets belong to them and are not part of a divorce. The defendant (divorcing couple) moved to strike out on the basis that the plaintiffs (a Swiss society) lacked the legal status to sue in AB. Although the plaintiffs were registered in Switzerland and were recognized entities under Swiss law, they were not recognized under Canadian law. Held: foreign individuals can sue, provided they are not enemy aliens

Corporations Act – unregistered foreign corps cannot sue regarding contracts carried out in ABLaw tends to support granting status where the entity in question is recognized as a legal or judicial person by the laws of its home jurisdiction

Looked back to status within Switzerland Supported by comity

Page 59: INTRODUCTION - University of Victoria - Web.UVic.caweb.uvic.ca/~lssweb/wp-content/outline_uploads/76... · Web viewDow Corning (2000) (BCCA) Facts: Class actions for negligence in

Gathering Evidence and Compellability of WitnessesIn common law, most evidence is matter of procedure rather than substance:

Competence and compellability of witnesses are matters of procedureBUT standards of proof may be substantive.

The means of proving a fact in court is procedural BUT what facts you have to show is substantive.

In re Cohen: statutes which establish presumptions (which facts must be proved) are considered to be substantive and not procedural.

BC courts cannot compel witnesses which are outside the jurisdiction of BC or force the production of documents outside of BC

Forum court needs to enlist the support of the foreign jurisdiction where the witness is or where the documents areFairly easy within Canada because there is legislation – Subpoena (Interprovincial ) Act:

Reciprocating provincial agreement – reciprocating province will accept a court order form BC compelling a witness to testify or produce documents. Therefore, within Canada can indirectly compel witnesses and documents indirectly through the legislation

Witnesses and documents outside of Canada are not covered by the legislation:

BC courts would then require judicial aid from the foreign courtTwo ways to seek judicial aid:

Apply directly to the foreign courtRequest the BC court to issue letters rogatory (more common):

Request the foreign court’s assistance in the production of documents or obtaining oral examination Even if don’t get witness to come to BC, may be able to do or an oral

examination or see the documents in the foreign jurisdictionDifficult to get the foreign court to actually order a person to go to BC, but helpful to get aid of foreign courtDiscretionary on the BC court to issue the letter rogatory, and also discretionary on the foreign court to oblige - largely a matter comity

Canada is member to a number of treaties dealing with evidence taken abroad:

May provide for the possibility of an officer from a consulate of embassy taking evidence from a foreign person or documentGiven the international nature of the world, need for this kind of arrangement

Blocking LegislationSome provinces have produced legislation prohibiting the production of documents to foreign jurisdictions

Page 60: INTRODUCTION - University of Victoria - Web.UVic.caweb.uvic.ca/~lssweb/wp-content/outline_uploads/76... · Web viewDow Corning (2000) (BCCA) Facts: Class actions for negligence in

Hunt v. T. and N Plc (1993) (SCC)Facts: Quebec Business Concerns Record Act prohibited the removal of documents to another jurisdictions at the request of a foreign court order.Held (La Forest): Whole purpose of the statute was to impede successful legislation ran afoul of the constitutional mandated requirement to give full faith and credit to sister provinces. Didn’t strike the legislation down, but merely read it down to be inapplicable between Canadian provinces.

NOTE: Full faith and credit goes beyond judgements and law and also extends to court orders and discovery

More difficult issue when dealing with a jurisdiction outside of Canada: Canadian courts are reluctant to allow foreign rules relating to production of evidence to be argued to get around Canadian production rules.

Ed Miller Sales (1988) (AB CA)Facts: Consent agreement between two individuals that a disposition is confidential. Witness in BC claims that they cannot be a witness because it will breach the consent agreement under Bahamian law. Had a court order compelling them to testify, but said that testifying would violate Bahamian law. Held: while comity had to be respected and the Bahamian law should not be disrespected, the Canadian court has the authority to compel testimony if the witness is in Canada

Canadian court is not likely to let foreign laws prevent them from ordering disclosure (if the witness or documents are in BC)

Flipside: if a foreign court comes to the BC court requesting assistance in a foreign action:

BC Evidence Act – allows someone to request the BC court to assist in the production of documents or witness testimony to a foreign courtHighly discretionary on the court to grant the order:

Will be very conscious of comity Must be a formal request, usually from a court Usually needs to be a case pending before the foreign courtMust have substantial case - can’t simply be a fishing evidence (but a difficult line to draw)

Presbyterian Church of Sudan v. Rubiak and TalismanFacts: Church brings action against Talisman for assisting the genocide in Sudan. New York court issues an order to compel testimony from Rubiak, who was a human resources manager in Sudan. Held: Court looked at the nature of the case, the nature of the witness and the nature of the evidence.

Brought mainly on the fact that a New York court order had been issuedRubiak was not a major decision maker; Plaintiff had already interviewed countless other witnesses and had not gained sufficient evidence for their case.

Page 61: INTRODUCTION - University of Victoria - Web.UVic.caweb.uvic.ca/~lssweb/wp-content/outline_uploads/76... · Web viewDow Corning (2000) (BCCA) Facts: Class actions for negligence in

Rationale for the letter rogatory was largely a fishing expedition did not provide sufficient evidence of the scope of the evidence they were seeking from Rubiak’s testimony. Therefore, request was denied.

Gulf Canada (1980)Facts: Court faced with a letter rogatory from the US Federal Court for the production of documents by Gulf Canada. Actual case was the Gulf Oil Company v. Gulf Canada (Canadian subsidiary of Gulf Oil)Held: Court said that it was against the public policy of Canada to release the documents.

Nature of the court case was perceived to be that through US legislation, the US was essentially directly reaching into Canada to tell one company not to do what they were legally entitled to do in Canada

With this type of long armed legislation, the court will not enforce the request as a matter of public policy.

Ministerial DiscretionThe Foreign Extraterritorial Measures Act gives the Minster of Justice the capacity to issue an order denying or preventing a court from accepting letters rogatory from a foreign court – allows the Minister to trump a court decision to compel witness testimony or production of documentsDesigned to protect against long-armed reach of foreign law trying to interfere with Canadian companies engaging in activities which are perfectly legal in Canada

Public Policy ConsiderationsNo BC court want to apply foreign law which inconsistent with Canadian public policy

In box three, may argue that applying foreign law is against public policyIn box four, may argue that enforcing the foreign law may be against public policy

Public Law ClaimsAs a matter of choice of law, you will not include foreign revenue law, penal law or generally described public law. This also works for the enforcement of law: will not enforce revenue law, and probably not penal law

General rule: public policy and public law are pleaded as exceptions to the application of the foreign law.

Public policy does not create choice of law. Rather, public policy is argued at the end to argue against the application or enforcement of the law.

Issues meeting the public policy bar:SlaveryNoxious drugs for non-medicinal uses

Page 62: INTRODUCTION - University of Victoria - Web.UVic.caweb.uvic.ca/~lssweb/wp-content/outline_uploads/76... · Web viewDow Corning (2000) (BCCA) Facts: Class actions for negligence in

Gambling

USA v. Ivey (1990) (OCA)Facts: Civil action in Michigan under US statute which allow US environmental authorities to recover funds for environmental cleanup. Ivey was an officer in the defendants company who lived in Ontario but did not want to testify in Michigan. Ontario court was asked to enforce a Michigan court order against Ivey to testify.

Arguments: Up to the lex fori to determine how to characterize the foreign legislation. Thus, the Ontario court can characterize the nature of the foreign law.

Penal law: the law that is used is US federal penal law. Court notes that there is general rule against recovery for penal laws. No question that the penal law exception exists.

Held: found that the US scheme was about reimbursement, not penal law – did not fit the definition. Only used to reimburse and compensate for what the company had an obligation to do (clean up), not a punishment or directly designed as a deterrent

Revenue law: argued that the underlying purpose of the US statute was a site specific taxation scheme.

Held: Ontario court called this ungrounded rhetoric – not revenue law. Although the nature of the law was not precisely defined, it was not made to raise money for government, but rather was for reimbursement for environmental damage.

Public policy: argued that the severity of the US legislation as compared to the Ontario legislation should render the decision based on the Michigan legislation unenforceable would be against Ontario public policy

Murky area - established to some measure in certain jurisdictions, but hasn’t got full recognition Held: Court notes that the remedies required in the Michigan legislation were not that different from what was required under Ontario legislation. It is not enough that the local law on the same point differs from the foreign law, fundamental values must be at stake (more than just a mere difference in policy)

Disparity between the two regimes was not sufficient to meet the public policy testSimilarities far outnumbered the differences between the two legislations

Boardwalk Regency: Public policy is an exception to the choice of law rules – if the foreign law is inconsistent with public policy, then it should not be enforced

Very high threshold to use this exception, especially between sister provinces – must give full faith and credit not only to court judgements, but also the laws of sister provincesCastelles: Foreign law must violate some fundamental principle of justice, some prevalent conception of good morals, some deep rooted tradition of the commonwealth…”

Page 63: INTRODUCTION - University of Victoria - Web.UVic.caweb.uvic.ca/~lssweb/wp-content/outline_uploads/76... · Web viewDow Corning (2000) (BCCA) Facts: Class actions for negligence in

Natural Justice: argued that enforcing the judgement would violate natural justice

A lack of natural justice is a defence to the recognition and enforcement of a foreign judgementGranting judgment without any notice to the defendant or a complete lack of opportunity to defend yourself could = breach of natural justiceHeld: the rules of liability and burden of proof are matters for the foreign jurisdiction, there is no authority for non-enforcement of a judgement merely because it is rendered on the basis of strict statutory liability.

Requires a fundamental deviation from forum natural justice values it is insufficient to show simply that the forum and foreign procedures diverge

Court notes that natural justice exists in theory, but has rarely been successfully utilized in Canada. Cites Beals v. Saldana:

Majority: held that natural justice was not applicableDissent: supported the non-enforcement of the decision based on the lack of natural justice had not had proper notice of the case (used natural justice as one reason why they would not enforce the award)

Held: All of Ivey’s arguments failed enforcement of the Michigan decision was allowed

Kuwait Airways v. Iraqi Airways (HoL) (2002)Rare case: Court did uphold that the law to be applied was inconsistent with UK public policyFacts: During the first Gulf War, the Iraqi government expropriated the aircraft in Kuwait. Following the end of the war, the aircraft were returned to Kuwait. Kuwait Airlines then brought an action against Iraqi Airlines in torts for taking their aircraft.

British choice of law for tort is different than in Canada: to bring action in the UK for tort outside of the UK, the cause of action must be actionable both in the UK and in the place where it occurred

Tort occurred in Iraq. Under Iraqi law, the expropriation was valid. Therefore, no actionable tort would arise in the UK. Moreover, title to tangible property in choice of law is determined by where the tangible property is located – lex situs was located in Iraq. Again, no cause of action in the UK.Hence, the law to be applied is Iraqi law

Argument: Since the Iraq law was to be applied, and there was no cause of action under Iraqi law, Kuwait Airlines relied on the public policy argument to show that the Iraqi law shouldn’t be applied

Reasoning: Mere difference between foreigh law and the forum law is not sufficient to trigger the public policy defence.

“Blind adherence to foreign law can never be required of an English court. Exceptionally and rarely, a provision of foreign law will be disregarded

Page 64: INTRODUCTION - University of Victoria - Web.UVic.caweb.uvic.ca/~lssweb/wp-content/outline_uploads/76... · Web viewDow Corning (2000) (BCCA) Facts: Class actions for negligence in

when it would lead to a result wholly alien to fundamental requirements of justice as administered by an English court. A result of this character would not be acceptable to an English court. In the conventional phraseology, such a result would be contrary to public policy. Then the court will decline to enforce or recognise the foreign decree to whatever extent is required in the circumstances.”

Argument: Iraqi Airways argued that this is the only time that public policy will prevail is if there is a gross violation of human rights

Court held that gross violations of human rights would certainly allow public policy to prevail, but rejected this as the only grounds to trigger public policy

Argument: Iraqi Airways argued that if expropriation was in violation of public international law, this was not in itself inconsistent with public policy

HoL accepted that argument: test is not whether there is a breach of public international law. However, this is merely a consideration, and is not determinative

Importance of context:Public policy to be determined by contemporary standards in the context of the Gulf WarInternational law violation by the Iraqis in invading Kuwait – this war would not be recognized by the UK “Enforcement or recognition of this law would be manifestly contrary to the public policy of English law… it would sit uneasily with the almost universal condemnation of Iraq's behaviour and with the military action, in which this country participated, taken against Iraq to compel its withdrawal from Kuwait.”

Comment: If had been straight up expropriation minus the war, might get a different result

Boardwalk Regency v. Marouf (1992) (OCA)Facts: Gambling debt incurred in New Jersey by an Ontario resident. Casino obtained a default judgment in New Jersey and sought to enforce the judgement against Marouf in Ontario. Ontario statute provided that gambling contracts were rendered void and unenforceable.

OCA and lower court recognized that the proper law of the contract was New Jersey law. However, the Ontario statute did not have extraterritorial application – didn’t say that contracts made outside of Ontario were void (only applied to gambling contracts in Ontario). Thus, gambling contracts in NJ not covered by the Ontario statute.

Issue: whether the enforcement of the gambling contract was contrary to the public policy of Ontario.

Rule: If you incur gambling debts in other jurisdictions, they can be enforced against you in Canada

Page 65: INTRODUCTION - University of Victoria - Web.UVic.caweb.uvic.ca/~lssweb/wp-content/outline_uploads/76... · Web viewDow Corning (2000) (BCCA) Facts: Class actions for negligence in

Held: Public policy argument rejected – gambling contract enforced.Court noted that both Ontario and NJ regulated gambling and gambling debts – determined that the non-enforcement of gambling contracts can’t be against public policyMust be more than the morality of some people and run through the moral fabric of society

Dissent (Arbour): Refused to accept that it would not be breach of public policy to allow the foreign law to apply within Ontario.

While some gaming is okay in Ontario, the type of gaming in this case was in fact criminal in Ontario (at the time).To allow enforcement of the gambling would essentially mean that the Ontario law is conceding to foreign law:

Money was advanced to the respondent for the purposes of engaging in gaming which in Ontario would be criminalWould not be sound public policy to allow the enforcement as it would be yielding to foreign law

Comment: This dissent has never been followed

Society of Lloyds v. Saunders (2001) (OCA)Facts: Lloyds Insurance “names” those folks who provide the money to back up the insurance claims that can be made against Lloyds through very odd contractual arrangements. The contracts allow for the Lloyds syndicate to call on letters of credit that are placed at the disposal of the syndicate by the “names”.Insurance industry hits bad times. UK judgments came down requiring “names” to pay up.

Saunders has to pay up to his letter of credit, but wants to get out.Agreements between Lloyds and the “names” were solicited in Ontario, but the solicitation did not comply with the relevant Ontario securities regulations. Contracts in question had a choice of forum clause and a choice of law clause: England. Key fact: there had been a case brought in Ontario by the names against Lloyds and the OCA had stayed that case the choice of forum and choice of law clause meant that the proceedings should properly be held in the UK.

Argument: Saunders argues that the UK judgment should not be enforced in Canada because it would be against public policy

Reasoning: court reviews cases trend of jurisprudence is that the public policy exception is to be narrowly construed and rarely applied. OCA decided that public policy does involve more than just morality issues However, the exception is still very narrow.

Notes that in the environment where the courts are to respect international comity more frequently, public policy need to play a larger role that was previously the case “That trend is to emphasize the concept of comity among nations and particularly among provinces of this country when addressing the issue of enforcement of judgments and choice of law.”

Page 66: INTRODUCTION - University of Victoria - Web.UVic.caweb.uvic.ca/~lssweb/wp-content/outline_uploads/76... · Web viewDow Corning (2000) (BCCA) Facts: Class actions for negligence in

Refers to Morguard and Tolofson: “In both cases, the role of the public policy concept was left, in effect, as a safety valve to prevent anomalies.” broadening of the public policy exception

In this case, the public policy issue is (para. 68): “Because the primacy of the protection of our capital markets and the role that the full, true and plain disclosure obligations and the anti-fraud measures contained in securities regulation legislation plays in the scheme for that protection is basic to the well-being of our economy and our society, it is, in my view, beyond dispute that taken on its own and in isolation from other factors, to condone a breach of those obligations would be contrary to the public policy of Ontario. However, to view the disclosure obligation provisions of the Securities Act, such as the prospectus requirement, as akin to a moral imperative may be to stretch the concepts unnecessarily. Public policy has been universally described as "fundamental values" and "essential principles of justice". In my view, it is appropriate at this stage in the development of our society, to characterize the protection of our capital markets and of the public who invest in and depend on the confident and consistent operation of those markets as such a fundamental value.

Held: Okay in this case to enforce the English judgement. While the disclosure provisions are of fundamental value, there was in the end no public policy objection.

Fundamental though is the fact that the Ontario courts had already dealt with the issue in the earlier judgement and granted a stay of proceedings

Public policy may not turn exclusively on repugnance for the fundamental values represented by the underlying legal basis for the judgement, but on whether the forum courts are prepared to enforce a foreign judgement when an action on the same cause, had it been litigated in the forum, would not have been entertained Would have concluded that contrary to public policy but:

Previous case where Lloyds had been successful in getting a stay in Ontario – by refusing to hear the action, courts had contemplated that the result might not be in accordance with the Act If they had been concerned about enforcing the public policy than this

consideration would have trumped and they would have heard the case Don’t want to undermine the credibility of their own courts!

International comity – Lloyds is getting judgements in England against names all around the world Courts elsewhere have recognized the English court’s authority

to hear these cases and had been enforcing the judgements. If Ontario wanted to act weirdly, would cause unfairness, economic turmoil, etc.

Take home message: as you expand comity (the new ethos of conflicts), you also need to look at some of the defences in a new way

Page 67: INTRODUCTION - University of Victoria - Web.UVic.caweb.uvic.ca/~lssweb/wp-content/outline_uploads/76... · Web viewDow Corning (2000) (BCCA) Facts: Class actions for negligence in

Beals v. Saldanha (2003) (SCC) Facts: Geoffrey and Leueen Saldanha and Dominic Thivy were residents of Ontario and sold lots they owned in Florida to Frederick and Patricia Beals. Beals brought an action against Saldanha and Thivy when it was discovered that the defendants did not actually own the property they sold. A defence was filed with the Court but did nothing afterwards and defaulted. A jury awarded Beals $260,000 in damages.

Thivy and Saldanha were informed by a lawyer in Ontario that the judgment could not be enforced and so they did nothing Default judgment in Florida.Soon Beals brought an action in Ontario to enforce the judgment, which had grown to $800,000 with interest.At trial the judgment was denied on the basis that the damages had been improperly assessed. On Appeal the Court allowed the foreign judgment.

Issue: whether the Florida judgement should be enforced against the family in Ontario.

Defences used in the case:Contrary to public policy of the forum – refer to previous discussion; cannot be based on foreign penal or revenue laws Judgement obtained by fraudJudgement obtained in breach of natural justice (both dissents in Beales) NOT a defence to say that the foreign court erred in law (because forum court is not a Canada!)

Majority: upheld the judgement on comity grounds

Defence of public policy – rejected:“This defence prevents the enforcement of a foreign judgment which is contrary to the Canadian concept of justice.  The public policy defence turns on whether the foreign law is contrary to our view of basic morality - the traditional public policy defence appears to be directed at the concept of repugnant laws and not repugnant facts…

It would prohibit the enforcement of a foreign judgment that is founded on a law contrary to the fundamental morality of the Canadian legal system.  Similarly, the public policy defence guards against the enforcement of a judgment rendered by a foreign court proven to be corrupt or biased. 

“The use of the defence of public policy to challenge the enforcement of a foreign judgment involves impeachment of that judgment by condemning the foreign law on which the judgment is based.  It is not a remedy to be used lightly.  The expansion of this defence to include perceived injustices that do not offend our sense of morality is unwarranted.  The defence of public policy should continue to have a narrow application.

Dissent (Lebel):What is being tested is the foreign law, not repugnant facts or outcomes

Page 68: INTRODUCTION - University of Victoria - Web.UVic.caweb.uvic.ca/~lssweb/wp-content/outline_uploads/76... · Web viewDow Corning (2000) (BCCA) Facts: Class actions for negligence in

Agrees with the majority that: this defence should continue to be, as the trial judge put it, “directed at the concept of repugnant laws, not repugnant facts” Furthermore, the defence of public policy has long been associated with condemnation of the foreign jurisdiction’s law.  To extend it to cover situations where there is nothing objectionable about the foreign law but, rather, a defect in the way the law was applied might send the wrong message, one that conflicts with the norms of international cooperation and respect for other legal systems underlying the doctrine of comity.In my opinion, there is more work for this defence to do.  It should also apply to foreign laws that offend basic tenets of our civil justice system, principles that are widely recognized as having a quality of essential fairness.  Among these, I would include the idea that civil damages should only be awarded when the defendant is responsible for harm to the plaintiff, and the rule that punitive damages are available when the defendant’s conduct goes beyond mere negligence and is morally blameworthy in some way.  These are basic principles of justice that are reflected in some form in most developed legal systems, although the particular form in which they are expressed may vary. 

Need to expand defences (Lebel): The category of foreign judgments that are prima facie enforceable in this country has been greatly expanded by virtue of the adoption of the Morguard test for foreign-country judgments.  The law as it now stands will admit a default judgment emanating from a forum that the defendant did not consent to and may have been connected to only indirectly or not at all.  This is a salutary development in our law on jurisdiction; if there are sufficient connections between the action and the forum, the judgment should not be shut out on the basis that the forum was inappropriate.  But the possibility that the judgment should be unenforceable for some other reason should be considered anew in light of this new context.  Castel and Walker, supra, have commented that if this Court confirms the application of the Morguard test to foreign judgments, “it would seem necessary to revise the defences . . . so as to protect persons in Canada who have been sued in foreign courts from the particular kinds of unfairness that can arise in crossborder litigation, and so as to prevent abuse from occurring as a result of liberal rules for the enforcement of foreign default judgments”

Comment: Ted believes that Lebel’s dissent will carry the day, especially for default judgments

Wende v. Victoria (1998)Facts: Woman dies intestate in 1986, but her niece petitioned to be the lawful heir to the estate. However, question of whether the deceased had maintained a legal relationship with her family. In 1951 (at age 43), she was adopted by a German count with whom she was romantically involved and with whom she had been living for 12 years before the adoption. The count died in 1952 at age 84. Evidence was

Page 69: INTRODUCTION - University of Victoria - Web.UVic.caweb.uvic.ca/~lssweb/wp-content/outline_uploads/76... · Web viewDow Corning (2000) (BCCA) Facts: Class actions for negligence in

submitted that under German law, adoptions were considered as a means for legalizing the life partnership of an aristocrat and a person without noble birth. Under German marriage law, had prohibited marriages between persons so different in age (repealed in 1946). No evidence that couldn’t then have been married in 1951. Under the BC Adoption Act s. 47, there was a choice of law clause to determine the validity to the adoption: look to where the adoption took place – if valid there, then valid here. If the adoption is valid, then this severs the connection with the original family – niece would be out of luck and the province would get the estate.

Issue: Niece argues that the adoption should not be recognized for reasons of public policy

General position in BC: if the adoption is legal in the place where it took place, then it will be considered a legal adoption in BC – statutory choice of law.

Court noted that is was very rare that a foreign adoption would not be recognized in BC

Held: Adoption not recognized on the grounds of public policy – not intended to establish a parent-child relationship, but rather to manage an estate and to allow the couple to continue to live together as husband and wife.

Comment: Court exercised unusually broad discretion in denying the adoption in order to get what they considered to be a fair judgment – niece gets the estate.

Penal LawChoice of law has no application to criminal law – courts always apply their own criminal law and do not enforce the criminal law of other states.

Really just a public policy issueForum court will only apply criminal law for event happening within its own territory

ContractsCriminal activity: unlikely to enforce a contract that is illegal where it was performed. That is the extent to which we import foreign criminal law into contracts

Spencer v. R.Canadian bank official objected to giving evidence because it would be a crime in the Bahamas (breach of confidentiality).Held: tough luck – you’re here, so testify! Forced the bank official to testify.

Huntington v. Attrill (1893) (PC)Facts: New York company issued false statements involving company stock. Under NY law, each officer of the company who signed the document was made personally liable by statute. A suit was successfully brought against the bank

Page 70: INTRODUCTION - University of Victoria - Web.UVic.caweb.uvic.ca/~lssweb/wp-content/outline_uploads/76... · Web viewDow Corning (2000) (BCCA) Facts: Class actions for negligence in

director personally. Come to Ontario (where the director now lives) to enforce the judgement.

Argument: Director argues that unenforceable because this is NY penal law.General Rule: no proceeding which has the objective of punishment ought to be admitted for enforcement by the courts of another country

Forum court makes the determination: Ontario gets to decide if this is penal law or not.Although the opinion of the foreign court may be persuasive, the characterization of the law in the foreign jurisdiction is not determinative!

“The rule had its foundation in the well-recognized principle that crimes, including in that term all breaches of public law punishable by pecuniary mulct or otherwise, at the instance of the state government, or of some one representing the public, were local in this sense,-that they were only cognizable and punishable in the country where they were committed. Accordingly no proceeding, even in the shape of a civil suit, which had for its object the enforcement by the state, whether directly or indirectly, of punishment imposed for such breaches by the lex loci, ought to be admitted in the courts of any other country.”

Draws a distinction between laws that create a civil remedy that are protective and remedial in nature and those that are punishment provisions.

Held: provisions were protective and remedial, not penal able to enforce

Recall Ivey: court said that this was not a matter of penal law, merely reimbursement for recovery and cleanup of environmental damage

Revenue LawGeneral rule: a forum court will not enforce foreign revenue laws – taxes!

Again, really a public policy exception.Downside – does encourage tax flight.

BUT tax treaties have changed the playing field significantly, so many of the following cases have limited applicability:

Domestic and international treaties have made significant changes regarding the collection of taxesCanada and the US have agreed to recognize each others tax claims where certain conditions are metProvincial tax laws generally allow for the enforcement of tax claims from other provinces

Rationale: full faith and credit, comity, Morguard, etc.

Remember: the lex fori still gets to decide if tax law or revenue law.

USA v. Harden (1963) (SCC) – leading caseFacts: California wanted to recover taxes from someone in BC. Got a judgment in California and came to BC to enforce it.Issue: Should someone who incurs huge tax debt be able to hide in BC?

Page 71: INTRODUCTION - University of Victoria - Web.UVic.caweb.uvic.ca/~lssweb/wp-content/outline_uploads/76... · Web viewDow Corning (2000) (BCCA) Facts: Class actions for negligence in

Argument: California argued that this is not the enforcement of revenue law, but rather the enforcement of a California judgement - no one is asking BC to apply California law, but merely to enforce a California judgment.

General Rule: the forum court will not enforce foreign revenue laws – territorially based (improper extension of the taxing state’s authority)

Given the generality, seems to encourages the idea of tax havens

Held: the rule that you cannot recover foreign tax debts was sufficiently well established and with such sound reasoning that it should not be circumvented by allowing enforcement of a judgment based on foreign revenue law.

Vandenmark v. Toronto Dominion Bank (1989) (Ont. S.C.J.)Facts: The IRS issued an order requiring the TD New York office to seize all of Vandenmark’s assets, including bank branches in Canada (Note: IRS order, not a court order). TD bank immediately caved and seized all of the money. Vandenmark alleged that money in Canada actually belonged to his parents.

TD cited the US freezing orders: vested interest – if they released the money, under US law they would be subject to a 200% fine

Held: TD seizure of assets in Canada was an indirect enforcement of US revenue laws.

“In any event, while acceptance by the bank of a penalty imposed in the United States might seem to be a hardship, the effect of permitting the Ontario branches to defend the applicants’ claim on the basis of the bank’s liability in New York State would be to enforce indirectly a claim for taxes by a foreign state and one that has, so far as the evidence discloses, not even given rise to a New York or Federal Court judgment. (i.e. Only an IRS order!)

Comment: has been changed by tax treaties

Stringam v. Dubois (1992) (AB CA) Facts: testator died in the US, left farm to niece in AB, executor wanted sale of the farm to pay Arizona estate tax.Issue: whether the rule against courts enforcing a tax claim of a foreign jurisdiction applies so as to allow transfer of the Canadian realty to the devisee, rather than requiring that the property be sold and the proceeds used firstly to pay the US estate taxesHarden rule applicable even where there is no direct involvement by the foreign stateReed – BC estate case where the CA had not applied Harden.

Limited to its facts: treasury of the foreign state was going to be directly enriched, foreign state was directly involved in the action to get the moneyWhat you can’t do directly, you really can’t do indirectly!

Held: farm not sold because it would be an indirect enforcement of Arizona revenue laws

Page 72: INTRODUCTION - University of Victoria - Web.UVic.caweb.uvic.ca/~lssweb/wp-content/outline_uploads/76... · Web viewDow Corning (2000) (BCCA) Facts: Class actions for negligence in

Comment: fairly strict rule that doesn’t seem to make sense (because people moving all around the world, essentially allowing tax havens)

Harden has been criticized as too narrow, too bold, doesn’t take into account comity (decided before Morguard), way of business has changed Niece is actually an American resident – bank or executor had recourse against her if there was non-payment may distinguish this case on the factsAB CA suggests that SCC may want to re-examine this rule! Upheld this case on principle only

Re: Sefel Geophysical (1989) (AB QB) Facts: trustee of a bankrupt company applied for advice as to the appropriate distribution among creditors of the proceeds of sale of the bankrupt estate Issue: were revenue claims from the US and the UK barred by the tax exclusion? Held: Foreign claims are enforceable - court basically ignored cases that revenue claims in a bankruptcy are not enforceableBecause of international comity, not satisfied that the old English case is applicable anymore suggestion that some foreign tax claims should be recognized

“If the goal is to deal with liquidations in an orderly fashion in one country by virtue of deference shown by competing nations, surely some claims should at least be recognized. I am not dealing with the priority of those claims at this point, but rather I am saying that current comity principles suggest that some foreign tax claims should be recognized in a Canadian liquidation setting. Comity is about respecting foreign judgments, proceedings and acts of state. If our bankruptcy proceedings are respected and deferred to, as they were in the case at bar, I am of the opinion that the claims of foreign states should be respected in our proceedings as long as they are of a type that accords with general Canadian concepts of fairness and decency in state imposed burdens.”

BUT exception is restricted to liquidation proceedings (i.e. very narrow)!

ContractsMany conflicts cases arise in the area of contracts. There are many juridical sub-categories in contracts

There are five possible connecting factors in contracts:Where the contract was made – lex loci contractusWhere the contract was or is to be performed – lex loci solutionsWhere the parties reside or have main places of business – lex domiciliiWhere the subject matter of contract is located – lex re sitiiWhere the case is being brought – lex fori

Choice of law in contract is determined by judge-made rules, but beware when dealing with certain types of contracts!

Immovables land

Page 73: INTRODUCTION - University of Victoria - Web.UVic.caweb.uvic.ca/~lssweb/wp-content/outline_uploads/76... · Web viewDow Corning (2000) (BCCA) Facts: Class actions for negligence in

Warsaw Convention (air carriage), Hague-Visby rules (marine carriage), UN Convention on contracts for the International Sale of GoodsExcluded: arbitration agreements, matrimonial property rights, wills, etc.

Internationally uniform choice of law rules may apply (next best thing to uniform rules)

Rome Convention on the Law Applicable to contractual obligations: applies to any case brought before a court of a state party, whether or not the other country associated with the case is also a party to the convention, as long as there is a contractual obligation involving a choice of law issue OAS Convention: expressly limited to use between contracting parties, but the law chosen can be that of a non-contracting party

Are arbitrators bound by the same choice of law rules as the courts? International arbitrations no because Model Law article 28(2) says that the arbitration tribunal should apply the laws determined by the conflict of law rules it thinks are applicable There is no provision for inter-provincial arbitration arguably have to conform with domestic choice of law rules

Proper Law of ContractPrinciples:

Courts involved with contracts where conflicts are involved seek to uphold the contractCourt generally likes the autonomy of the parties - give the parties the maximum choice regarding choice of law. Really want to apply a choice of law clauseCertainty of commerce – courts don’t want to upset the business community by changing the law

When it comes to choice of law, the courts are looking for solutions that foster economic development – what is reasonable, what was contemplated by the parties, etc.

Definition of proper law of contract (Castelle, based on Dicey and Morris):The system of law by which the parties intended the contract to be governed or, where the intention is neither express nor to be inferred from the circumstances, the system of law with which the contract has the closest and most real connection.

Three possibilities for determining choice of law:What was expressly agreed upon as being the governing law: if there is a clear and unambiguous choice of law clause, then that clause stands

However, there are still ways to get around a clear and unambiguous choice of law clause, although much more difficult

If no explicit choice of law clause, look to see if the parties impliedly agreed on a choice of law intent of the parties.

But what do you look for? Choice of forum clause can be an indication

Page 74: INTRODUCTION - University of Victoria - Web.UVic.caweb.uvic.ca/~lssweb/wp-content/outline_uploads/76... · Web viewDow Corning (2000) (BCCA) Facts: Class actions for negligence in

If you can’t figure out the intention, then look for the real and substantial connection between the elements of the contract and the law of a jurisdiction

Similar in many ways to implied intent

Approach: 1. Look to see if there is express agreement (i.e. choice of law clause in

the contract)2. If not, can the parties be seen to have impliedly agreed to a choice of

law? 3. If not, look to what the contract is about (and to some extent also to

the people involved) and decide where the most real and substantial connection lies.

Note: US courts do not play a very important role in choice of law because they generally just go to real and substantial connection when there is no express choice of law clause

Vita Foods v. Unus Shipping Co. (1939) (PC)Facts: Plaintiff had a lot of fish in Newfoundland and wants to get the fish to New York. Plaintiff is a New York company who contracts with a Newfoundland shipping company to ship the fish to New York. Ship runs aground and the herring go bad. Plaintiff comes to Nova Scotia to bring action because that is where the defendant has assets. No question that the captain of the ship was negligent.

Bill of lading (contract between the cargo owner and the shipping company)

Contract signed in NewfoundlandContract states that the vessel owner would not be liable for any negligence whatsoeverChoice of law clause: states that English law will apply

Problem: The bill of lading that was signed did not bring in the Hague rules, which balance the liability between the owner and ship under Newfoundland law, the Hague rules had to apply! Thus, under Newfoundland law, bill of lading might be invalid!

Important because if you can kick out the bill of lading in its entirety, then you can kick out the no negligence provision!

Arguments:Vita Foods argues that because the bill of lading didn’t incorporate the Hague provisions and thus didn’t follow Newfoundland law, the contract was invalid and the common law should governUnus Shipping argues that the bill of lading is still valid: the no negligence clause stands and the choice of law is England.

Issue: Whether the contract bill of lading was valid or not because of its failure to comply with the Newfoundland statutory regime.

If the contract is not valid, the common law applies regular negligence rules. If the contract is valid, the no negligence clause stands.

Page 75: INTRODUCTION - University of Victoria - Web.UVic.caweb.uvic.ca/~lssweb/wp-content/outline_uploads/76... · Web viewDow Corning (2000) (BCCA) Facts: Class actions for negligence in

Illegality of the contract:

Which law deals with illegality of contract – whose law determines that?PC held that you didn’t look to Newfoundland to determine if the contract was illegal or not

What is the importance of the choice of law clause?The parties expressed intention with respect to choice of law must be given effect express wishes must be respected to foster economic activity“In questions relating to the conflicts of law, the rules cannot be stated in absolute terms, but rather only as prima facie assumptions. Where there is an express statement by the parties of their intention to select the law of the contract, it is difficult to see what qualifications are possible, provided the intention is expressed is bona fide and legal, and provided there is no reason for avoiding the choice on the grounds of public policy.

If you’ve got an express clause, its going to be pretty hard to shakeNot necessary for the chosen law to have any specific connection to the facts, the parties of the case in general as long as it was a bonefide choice of law

Rule: The only exceptions to an express choice of law clause would be: Where there is a lack of bonefides in the selection of the law OR Where applying foreign law would be against the public policy of the

forum

In this particular case, the court noted that the parties may have wished to choose a law that is well developed in the area, and England had well developed shipping laws.

Insurance companies also likely in EnglandMaritime law ultimately has a large British component

Held: English law applied. The express choice of law clause did win out – Privy Council was unwilling to undermine that choice of law clause.Here the choice of law clause was clear – complications related to the legislation and the formation of the contract largely ignoredBut then up to the English law to determine what effect to give to the Newfoundland statute?

In this case, court stated that as a matter of policy the English court cannot be overly concerned with illegality under Newfoundland law. (Remember that it is 1939, and England did not give much effect to foreign law)

The fact that the bill of lading was issued inconsistent with the Newfoundland law did not make the contract illegal, and it is therefore not illegal in England

But PC noted that if the case had been heard in Newfoundland, you might have gotten a different result probably would have found the contract to be illegal

Page 76: INTRODUCTION - University of Victoria - Web.UVic.caweb.uvic.ca/~lssweb/wp-content/outline_uploads/76... · Web viewDow Corning (2000) (BCCA) Facts: Class actions for negligence in

However, finding of what a Newfoundland court would have done is largely irrelevant: perhaps persuasive, but certainly not binding!

Result: Contract was valid carrier exemption from negligence applied

Comment: express choice of law may not resolve all of the issues of the contract

Star Texas (1993) (Eng. CA)Significance: Stands for the proposition that if you have an arbitration clause or choice of forum clause that takes you a particular state, there is an inference that it carries with it a choice of law(even though this is what they didn’t do in this case)

Facts: An English company owns the ship and charters it to a Chinese company to ship chemical. Chemicals spill on board the ship and cause damage. English company brings suit in England. No choice of law clause in the contract, but contract contained an arbitration clause stating that any dispute is to be arbitrated in London or Beijing, depending on the choice of the defendant (if you get sued you choose the forum). But which court gets to interpret the arbitration clause and whether its even valid.

Arguments:The Chinese company challenges the ability to bring this action in a British court – supposed to go to arbitration argue that the English Arbitration Act allows them to challenge the jurisdiction of the English courtPlaintiff argues that the arbitration clause in the contract should not be given effect – ambiguous, poorly drafted, etc. Therefore, case should go to the English court.

Issue: Whether the arbitration clause, because of its relationship to choice of law, was rendered without effect.

Does a choice of forum clause raise an implication or lead to an implied agreement as to the choice of law regarding the contract. Was the arbitration clause null and void or void for uncertainty?

Basic argument: the choice of law is wherever the matter is arbitrated.

Held: Arbitration clauses (and maybe even choice of forum clauses), in the absence of an express choice of law clause, may give rise to an inference that there is an implied agreement as to the choice of law.

But if you haven’t clearly chosen the place of arbitration, you have not chosen the choice of law!In this case, the parties probably never thought of choice of law Decided that they wanted to go to arbitration, but no one decided whose law was going to apply!

Rule: Where the arbitration clause allows for single situs, then the arbitration clause provides a strong indication of the choice of law

Page 77: INTRODUCTION - University of Victoria - Web.UVic.caweb.uvic.ca/~lssweb/wp-content/outline_uploads/76... · Web viewDow Corning (2000) (BCCA) Facts: Class actions for negligence in

But where there is more than one situs, then the indication that they intended a certain choice of law is considerable less strong

Held: In the absence of any implied intention, the court must objectively ascertain the most real and substantial connection

Here, they determined it to be the law of England.

Leading Canadian case on proper law of contract (though not necessarily the leading case in Canada):

Imperial Life Assurance Co. v. Colmenares (1967) (SCC)Facts: A Cuban national purchases life insurance in Cuba through an agent of the Ontario company. Cuban moved to the United States and then wanted to cash out on the insurance policy. Insurance company refused, saying that Cuban law at the time prohibited the surrender of the cash value of the insurance policy if you were living in the US. Ontario law did not have such a clause. Case brought in Canada. No express choice of law clause in the contract.

Issue: if Cuban law applies, no payout. If Ontario law applies, plaintiff gets the payout.

Arguments: Court looks at real and substantial connection:Insurance company noted that the contracts were prepared in Ontario and written in standard Ontario form. Payable in US dollars and the request for cash surrender had to be made to the Ontario office. Cuban argued that the documents were drafted in Spanish, became effective through delivery to the defendant in Cuba, Cuban agent and the insurance had to be authenticated in Cuba according to Cuban law.

Reasoning: Lay out a number of factors to be weighed in considering real and substantial connection:

Place of contractingPlace of performance (but where do you perform an insurance contract?)Language of the contract Place of residence or business of the partiesNature of the subject matter or its situs

Held: As a question of fact, the contract was formed in Ontario – not determinative, but persuasive

Insurance contract was based on a standard form contract drafted in Ontario according to Ontario lawDecision to accept the risk was made in Ontario

Comment: Case kind of skips over the consideration of an implied intention for a choice of law clause, but there really are three considerations.

Amin Rasheed Shipping v. Kuwait Insurance (1984) (HoL)Facts: Shipping company incorporated in Libya, carries on business in Dubai, seeking to sue Kuwaiti insurers in an English court.

Page 78: INTRODUCTION - University of Victoria - Web.UVic.caweb.uvic.ca/~lssweb/wp-content/outline_uploads/76... · Web viewDow Corning (2000) (BCCA) Facts: Class actions for negligence in

Issue: What is the proper law of the contract?

Reasoning: Court looks first at intent of the parties. If there is no such implied agreement, then look to the closest and most real connection to determine the proper law of contract.

Court is very clear that there are three approaches, but that implied intention and real and substantial connection are very similar

Relevant factors:Most important: No indigenous marine insurance law in Kuwait

No Kuwait statutes relating to marine insurance – this is a specialized area, complex contracts, ancient language, could not be governed by regular commercial law and could only be understood by reference to English law “Not good enough to suggest that English law could be persuasive…”

Use of an English formNationality of the partiesSterling Pounds was the currency of the contractUse of the English language in the contract

Held: English law applies.Even if Kuwait law was the proper law, could still borrow from English law

Mandatory Legislation and Illegality of ContractTwo ways to attempt to get around a choice of law clause

Attack the choice of law clause itself for not being bonefideApply the choice of law clause, then attack it for being contrary to public policy

Nike Infomatic v. Avac Systems (1979) (BC)Facts: BC company (plaintiff) enters into a distribution agreement with an Alberta company (defendant). Contract had an odd choice of law clause: contract to be governed by the laws of BC, BUT if any provision of the contract contravenes the law of the place of performance, then that provision is deemed to not be part of the contract. Contract to be performed in Alberta. Under Alberta law, distribution agreements of this type were supposed to be registered, but this was not done. Under BC law, there was not registration requirement.

Arguments:BC company argues that the first sentence of the choice of law wins the day: BC law is supposed to apply. The second sentence is severable or subordinate.Alberta company argue the contrary: expressed intention (BC law to apply) overridden by the fact that the contract was signed, performed and entered into in Alberta and should be governed by the law of that province. Further, the clause is ambiguous, and therefore the law of the forum

Page 79: INTRODUCTION - University of Victoria - Web.UVic.caweb.uvic.ca/~lssweb/wp-content/outline_uploads/76... · Web viewDow Corning (2000) (BCCA) Facts: Class actions for negligence in

should apply (BC). Also argue contrapreferentum if ambiguous, should be interpreted against the drafter.

Issue: Effect that should be given to the choice of law clause

Cites Vita Foods: express intent of the parties should be respected unless the expressed intent is not bonefide or results in the application of a law that is against the public policy of the forum

Quotes Dicey and Morris: “No court ... will give effect to a choice of law (whether English or foreign) if the parties intended to apply it in order to evade the mandatory provisions of that legal system with which the contract has its most substantial connection and which, for this reason, the court would, in the absence of an express or implied choice of law, have applied.”

How do you tell if the clause is bonefide or not?If the contract directs you to a law that was deliberately and clearly chosen in order to avoid the application of mandatory legislation in the jurisdiction that has the most real and substantial connection. Consider:

Absent the choice of law clause, what law would apply based on real and substantial connection? Is there mandatory legislation that would apply? Was the choice of law clause implemented in order to avoid this legislation?

Held: In this case, found that the choice of law clause stood, so BC law to apply

Without the choice of law clause, Alberta law would probably apply. Looks at whether there is a connection with BC:

Cites Vita Foods in which English law was applied because it was specified in the choice of law clause even when there was otherwise not a very strong connection to England.Here there is a connection with BC, and it was much stronger than what was allowed in Vita Foods defeats any argument about the non-bonefides of the choice of law clause.

Although there were very strong connections to Alberta, this did not amount to lack of bonefides and is not sufficient to overcome the choice of law clause.

Golden Acres v. Queensland Estates (1969) (Queensland)Facts: Suit brought by a “real estate agent” after the sale of land for the collection of their commission. Agent was not licensed under Australian law. Under Australian law, if you are not a registered real estate agent, you cannot sue to get the commission from the sale of land. However, the contract which gave the right to sell the land stated that “for the purposed of the agreement, contract shall be deemed to be entered into in Hong Kong. Argued that this was a choice of law clause.

Page 80: INTRODUCTION - University of Victoria - Web.UVic.caweb.uvic.ca/~lssweb/wp-content/outline_uploads/76... · Web viewDow Corning (2000) (BCCA) Facts: Class actions for negligence in

Held: The choice of law clause was inserted specifically to avoid the problems associated with an unlicensed agent collect commission under Queensland law choice of law clause overridden

Court noted that but for the choice of law clause, the law of Queensland would clearly apply and would clearly govern the contract. Such deliberate avoidance of Queensland law was seen as being contrary to the public policy of the forum

Comment: Australian court conflates public policy and lack of bonefides as being the same thing However, they are no! Really more accurately argued that this was a lack of bonefides

The facts of each case are different not every law that manages to avoid forum law will necessarily have lack of bonefides. However, this one deeply offended the judge.

Court assumed that the only reason to choose Hong Kong was to avoid Queensland lawMay need to show some legitimate reason for the choice

Decision turns on whether the state has mandatory legislation and whether it is being deliberately circumvented.

In Golden Acres, the perception was that the legislation was mandatory and that there was a circumvention of this legislation Queensland decides that its own legislation is mandatoryIn contrast, in Vita Foods, the Newfoundland legislation was held to not be mandatory England deciding that Newfoundland legislation was merely directory (hardly surprising!)

When is a contract illegal (i.e. inconsistent with statutory requirements)?Four ways that a contract may be declared illegal:

Under the proper law of contractGeneral rule: contract will not be enforced

Under the law of the jurisdiction where the contract is to be performedGeneral rule: unclear, but probably won’t be enforced

Under the law of the jurisdiction where the contract was done (entered and signed)

General rule: where contract was done not terribly relevantUnder the law of the forum

General rule: depends on the nature of the legislation

Limits on the Parties’ Freedom to Choose the Proper Law Statutory: court is bound by any statute of the forum that dictates what the law of k is to be or that invalidates parties’ agreement as to the governing law (as set out in Vita Foods)

Usually don’t invalidate choice of law altogether, but instead require a certain particular substantive rule to be applied to the k irrespective of the proper law directed by the parties

Fact that the k is valid or is drafted appropriately for one system of law but is invalid or improperly drafter under another system can be considered an indicator as to the most close and substantial connection

Page 81: INTRODUCTION - University of Victoria - Web.UVic.caweb.uvic.ca/~lssweb/wp-content/outline_uploads/76... · Web viewDow Corning (2000) (BCCA) Facts: Class actions for negligence in

Usually, rules of the lex fori apply to the k only where they are procedural

BUT: legislature can override this and require that a substantive law be applied, even to a foreign k May be argued that even where the legislature has not expressed its wish to have the rule apply to foreign ks, this should be inferred from the terms of the statute

Where k can be performed without doing anything illegal, but the parties actual intention is to perform in a way that contravenes the law of the state (e.g. smuggling) – the illegal purpose has been held to make the k unenforceable on ground of public policy

Note: Whether the contract is void, voided or voidable makes little difference in this context

Illegality Under the Proper Law of ContractRule: Contract is not enforceable if contrary to the proper law of contract, even if it would be legal in the lex fori cannot legalize the contract by going somewhere else!Note: the forum court will look at the statutory law of the foreign state and will indirectly enforce the foreign statutory law in the forum.

Illegality Under the Lex ForiIf legal by the proper law of contract, but illegal under the lex fori, two ways to deal with this:

Attack it on public policy groundsLook at the forum legislation to determine whether what is being done is truly illegal in BC

Does the legislation have extra-territorial effect? Or is it only intended to apply to contracts made in BC?

E.g. gambling cases: upheld the gambling contracts in Nevada even though gambling was illegal in Ontario at the time

Avenue Properties v. First City (1986) (BCCA)Facts: A real estate deal in BC for the purchase of property in Ontario Contract goes sour. Avenue Properties (BC company) bought property in Ontario from an Ontario company. Avenue Properties wanted out of the deal and the Ontario company sues Avenue in Ontario. Avenue comes to BC and seeks a decision that the contract is not enforceable.

Choice of law clause: the law to be applied is that of Ontario. Ontario seller had been trolling for business in BC and had not filed a prospectus as required under BC law. Because there was no prospecutus, this made the contract unenforceable in BC. Appears to be a valid contract in Ontario.

Issue: Whether the illegality of the contract and its unenforceability under BC law precluded recovery on the contract? Does the BC statute apply?

Note: Choice of law clause here is not being challenged.

Page 82: INTRODUCTION - University of Victoria - Web.UVic.caweb.uvic.ca/~lssweb/wp-content/outline_uploads/76... · Web viewDow Corning (2000) (BCCA) Facts: Class actions for negligence in

Court notes that it can apply forum law to a contract where:The enforceability of the contract is a matter of procedureCourt is compelled by statute to apply BC legislationThe failure to apply the forum statute would be inconsistent with public policy

Rule: even where the choice of law clause is not in issue, the court may still apply a provision of local law in preference to the foreign proper law of the contract where it is satisfied that it would be contrary to public policy to do otherwise

Whether a contract is illegal or not must be determined not based on what would have happened had the contract taken place in BC, but whether the legislation of the forum has extra-territorial application which makes a contract conducted outside the province illegal in BCIn other words, does the forum law apply irrespective of the choice of law clause or the proper law of the contract?

Tension points:There is a fundamental tension between the concepts that nearly all issues under the contract should be decided by the parties themselves and the obvious desire by legislatures to impose restriction on contracting parties for reasons of fairness, broad social policy or economic regulation.There is a conflict between autonomy of the parties and legitimate regulatory actions by the legislature

Held: the BC legislation expressly applied to solicitations within BC for the sale of land outside of the province!

This makes it different from the gambling cases! BC legislation had to be respected by the BC court to not do so would be inconsistent with the public policy of the forum. If the legislation is constitutionally valid, then this is one way to undermine a choice of law clause or the proper law of contract.

Held: BC law applies. Contract is unenforceable

Comment: The choice of law clause could operate, but it was overridden by the BC legislation. However, in Golden Acres, the choice of law clause had to be attacked directly to be overcome.

If Illegal Under Where the Contract was DoneIf the contract is valid under the proper law of contract, valid in the forum, but illegal in the jurisdiction where the contract was entered into and signed

Rule: where the contract is done is largely irrelevant

Vita Foods: Newfoundland legislation which perhaps made the contract illegal where it was made. Choice of law clause directed action to England. Forum court was Nova Scotia.

Page 83: INTRODUCTION - University of Victoria - Web.UVic.caweb.uvic.ca/~lssweb/wp-content/outline_uploads/76... · Web viewDow Corning (2000) (BCCA) Facts: Class actions for negligence in

Held: Newfoundland statute did not apply and did not matter, even though the contract was signed there. Lex loci contractus was not determinative of illegality of the contract. Comment: but if the Newfoundland court was the forum court, then they might have been bound by the legislation to declare that the contract was illegal if: the enforceability of the contract was procedural in nature, or the Newfoundland law had extra-territorial effect.

If Illegal Under the Place of PerformanceIf the contract is valid in the forum, but in illegal in the place of performance:

Gillespie Management v. Terrace Properties (1989) (BCCA)Facts: A claim for real estate management fees that arose in Washington State. Property owner was in BC. Property management company was in Washington. Manager of the Washington company was in BC. The property was in Washington. Contract was drawn up in BC. Property owner came to the BC court for breach of contract. But under Washington law, the manager of the property company had to be licensed as a real estate agent in Washington, which they were not. Washington law applied to both resident and non-resident property brokers (i.e. intended to have extra-territorial effect as long as had business in Washington). Onsite management was done in BC (by phone?).

BC was the proper law of the contract (not disputed). Contract is being performed in Washington.

Held: In order to support its claim for damages, the respondent would have to rely on acts that would have been illegal in Washington if they had been performed. The Washington state law provided that it was unlawful for any person to act as a real estate broker by negotiating in the lease or rental of real estate without first obtaining a licence. The claims for the performance of such illegal acts were not sustainable in British Columbia.

The forum court will not allow enforcement of a contract where it would be illegal in the place of performance, even if performing it in BC would have been legal. Forum court get to determine if the contract is illegal or not in the place of performance.

Southin (concurring): BC forum court should not enforce the illegality of the Washington contract because it would be contrary to public policy.

“The doctrine of illegality is founded on considerations of public policy - not foreign public policy but the domestic public policy of not enforcing unlawful bargains or requiring unlawful conduct. I leave open the question whether this Court must always defer to the law of the foreign state and hold that that which is unlawful there is unenforceable here as contrary to our public policy. But, as a matter of our own public policy, I think we should give effect, in these circumstances, to foreign legislation which is of the same order as domestic legislation.”

Page 84: INTRODUCTION - University of Victoria - Web.UVic.caweb.uvic.ca/~lssweb/wp-content/outline_uploads/76... · Web viewDow Corning (2000) (BCCA) Facts: Class actions for negligence in

Comment: Southin’s decision tends to complicate the matter by bringing in a very broad public policy argument.

Not the leading approach!

However, some academics have disagreed with the approach taken by the court in Gillespie Management:

Castelles: There is no valid reasons for Canadian courts to not apply domestic law of contracts where the forum is the proper law of the contract.Argues that illegality at the place of performance is irrelevant should only look to the proper law of contract. To do otherwise is to destroy the foundation of the proper law doctrine. Questions of illegality at the place of performance has nothing to do with this question Applying a strict choice of law analysis, the only relevant question should be: was the enforcement of the contract inconsistent with the proper law of the contract?

Comment: Don’t want to dismiss Castelles, but court will be wary of enforcing contracts where they are illegal where they are to be performed.

Contract FormalitiesThere are very few statutes that deal with the formalities of contracts

Greenshields v. Johnston (1981) (AB CA)Facts: Greenshields was an investment broker in Canada with offices in Toronto and Edmonton. Defendant was an Alberta company. Greenshields wanted payment on a commission from the Alberta company. Defendant company had a principle officer who had personal guarantee for payment of the commission.

There was a contract signed in Alberta, but Greenshields was mostly an Ontario company and the choice of law was that of Ontario.

Arguments:Alberta company argued that the contract was invalid because it did not comply with an Alberta statute that required the contract to be signed before a notary public. Ontario had no such law, and Ontario was the proper law of contract.

Which law applies to the formalities of contract?Rule: Contract is valid if it meets the formalities either of the place where the contract was made (lex loci contractus) OR by the proper law of contract.

Two connecting factors: As long as you meet one of the two, then the contract will be valid. Choice of law rules meant to ensure that the contract cannot be avoided based on a technicality.

Issue: How do you attack this?

Three argument attempted in this case, all of which fail:

Page 85: INTRODUCTION - University of Victoria - Web.UVic.caweb.uvic.ca/~lssweb/wp-content/outline_uploads/76... · Web viewDow Corning (2000) (BCCA) Facts: Class actions for negligence in

Attack the clause itself - whether the choice of law was a bonefide choice

Consider if there was an attempt to avoid mandatory legislation not so here, since Greenshields was mostly an Ontario company

Argue that the Ontario law should be ignored because its contrary to the public policy of Alberta

Court noted that there was significant connection to Ontario and therefore was not not likely to be inconsistent with the public policy of Alberta

Argue that the legislation is procedural and therefore the Alberta court must apply it.

Failed – legislation went more to substance.

TortsTollefson v. Jensen changed the choice of law for tort overnight!

RULE: Juridical category of torts: the relevant connecting factor is lex loci delecti – where the tort occurred

Unresolved questions:Does it apply to all torts?

Appears that lex loci delecti is was intended to apply to all torts and not simply car accidents, but there are exceptions (e.g. defamation) – could treat as a separate juridical categoryCanadian cases and commentators have tended to say that Tolofson applies to all torts – uphill battle to go against this

What does lex loci delecti cover?Castelles has a list (pretty much everything related to the tort – remoteness, contributory negligence, etc.)

Where does the tort arise?There are situations where the action occurs in one place but the consequences are felt somewhere else. In such a case, likely that the tort arises where the consequences of the action are felt.Difficulties also arise when the action arises from interprovincial or transnational transactions.

Tolofson v. Jensen (1994) (SCC)Facts: Father and son from BC in a car accident in Saskatchewan with another driver from Saskatchewan. The son was 12 at the time. Under BC law, he had to wait until he was 19 before he could sue, at which time he sues both his father and the Saskatchewan driver.

Issue: Under Saskatchewan law, for a passenger to be able to sue the driver you had to show wilful and wanton misconduct of the driver.

Lucas v. Gagnon (1994) (SCC) - companion caseFacts: Same facts, different provinces. Plaintiff is from Ontario, accident takes place in Quebec. Bring the case in Ontario against the father, who was the driver, and against the Quebec driver. Under Ontario law you can sue in tort both drivers. Under Quebec law, you cannot sue in tort, but can only achieve legislative benefits (no fault scheme).

Page 86: INTRODUCTION - University of Victoria - Web.UVic.caweb.uvic.ca/~lssweb/wp-content/outline_uploads/76... · Web viewDow Corning (2000) (BCCA) Facts: Class actions for negligence in

In both cases, determining the choice of law determines the outcome! If the Saskatchewan law applies, the son will not be able to recover (since can’t show wilful and wanton misconduct)If the Quebec law applies, no tort but will get legislative benefit

Held: SCC pretty much trashes the law as it existed at the time. Does a survey of the previous law on choice of law for tort:

McLean v. Pettigrew (1945) (SCC)Leading SCC case at the time

Facts: Quebec resident has a car accident in Ontario. Brings action in Quebec. But gratuitous passenger rule in Ontario would not allow liability (but would be liable in Quebec).

Held: SCC applies the old British rule:Regardless of where it took place, if the tort was actionable in the lex fori, and not justifiable where it occurred, then you could sue.

Rule redefined in Chaplin v. Boys:As long as the tort is actionable in both the lex fori and where the tort occurred, then you can sue.

American ApproachBabcock v. Jackson (1963) (New York CA) Facts: All of the parties from New York. Have an accident in Niagra Falls, Ontario and then bring action in New York. Discuss proper law of tort: Likes the approach of proper law of contracts and tries to apply it to tort choice of law governed by the forum that is most intimately concerned with the litigation.

SCC is highly critical of the development of the Anglo-Canadian case law:Notes that the British confuse jurisdiction simpliciter with choice of law; give an unduly strong role to the forum law of England.No reason to retain any of this stuff.

Held: From the general principle that a state has exclusive jurisdiction within its own territories and that other states must under principles of comity respect the exercise of its jurisdiction within its own territory, it seems axiomatic to me that, at least as a general rule, the law to be applied in torts is the law of the place where the activity occurred, i.e., the lex loci delicti.

Supports with practical considerations:“The rule has the advantage of certainty, ease of application and predictability. Moreover, it would seem to meet normal expectations.

Ordinarily people expect their activities to be governed by the law of the place where they happen to be and expect that concomitant legal benefits and responsibilities will be defined accordingly. The government of that place is the only one with power to deal with these

Page 87: INTRODUCTION - University of Victoria - Web.UVic.caweb.uvic.ca/~lssweb/wp-content/outline_uploads/76... · Web viewDow Corning (2000) (BCCA) Facts: Class actions for negligence in

activities. The same expectation is ordinarily shared by other states and by people outside the place where an activity occurs. If other states routinely applied their laws to activities taking place elsewhere, confusion would be the result. In our modern world of easy travel and with the emergence of a global economic order, chaotic situations would often result if the principle of territorial jurisdiction were not, at least generally, respected. Stability of transactions and well grounded legal expectations must be respected. Many activities within one state necessarily have impact in another, but a multiplicity of competing exercises of state power in respect of such activities must be avoided.

Leaving aside the British practice which is giving increasing… the practice of most states favoured lex loci delecti

Having adopted lex loci delecti, court turns to whether there are any exceptions:

Rejects the possibility of any exceptions within the internal Canadian contextPromotes settlement of disputes and within Canada the jurisdictions in Canada are similar enough that it should not create undue hardships.

Limited international exception Leaves the door open to tort occurring outside of Canada

One of the main goals of any conflicts rule is to create certainty in the law. Any exception adds an element of uncertainty. However, since a rigid rule on the international level could give rise to injustice, the courts should retain discretion to apply their own law to deal with such circumstances, although such cases would be rare. Indeed, if not strictly narrowed to situations that involve some timely and close relationship between the parties, an exception could lead to injustice.

Comment: consider the difficult fact pattern where all the parties from one jurisdiction but get into an accident in another jurisdiction

Results:Tolofson: plaintiffs lost – no recovery for injury from the accidentLucas: get some money out of the Quebec legislative benefit, but is not able to sue in tort

International Exception:Somers v. Fournier (2002) (ON CA)Facts: Action commenced in Ontario arising from motor vehicle accident in New York state between an Ontario resident and a New York resident.

Argument for an exception to the lex loci delecti rule:Application of the substantive law of NY would create serious injustice and hence fell under the basis for the international exception cases outlined in Tolofson.

Page 88: INTRODUCTION - University of Victoria - Web.UVic.caweb.uvic.ca/~lssweb/wp-content/outline_uploads/76... · Web viewDow Corning (2000) (BCCA) Facts: Class actions for negligence in

In Tolofson, SCC said courts could retain a discretion to apply the local law (lex fori) in international litigation where necessary to avoid injustice

Ontario CA says exception was not to be specialised, it should be limited only to compelling and exceptional circumstances

Court examines approaches taken by other courts: Hanlan v. Sorensky (OCA) - Court applied the international exception

Facts: Plaintiff injured as a passenger on a motorcycle in Minnesota. All parties from Ontario.Held: Ontario law governed since all parties were from Ontario.

Considered whether another legal system was more closely connected to the action. Decided that Ontario had a stronger connection than the lex loci delecti and that the court had discretion to apply local law.

Ontario residentsVehicle registered in OntarioInjustice would have arisen by applying Minnesota law

Wong v. Wei (1999) (BCSC)Facts: Plaintiff and defendant both BC residents. Motor vehicle accident in California. No liability on the part of any other parties. Plaintiff wanted to apply California law because no cap on damages. Defendant wanted BC law.Held: Court avoids substantive/procedural distinction and instead goes to Tolofson tort rule that the law to be applied is lex loci delicti interprovincially but internationally there is a small discretion to apply lex fori

Court exercises the discretion and applies lex fori, so no need to decide the cap issueCourt used an “unjust” test to decide which law to apply: California law would burden defendant with unlimited damages for non-pecuniary loss

Further, BC is the only jurisdiction of persons involved

Issue 1: In the application of the substantive law of NY, what aspects of the proceeding are substantive or procedural?

Held: Court looks at Tolefson and the distinction drawn between substance and procedure:

Substantive law is seen as creating rights and obligationsProcedural law is a means to meet those ends

Costs: proceduralUnder NY state law, costs are considered to be substantiveIn Ontario, costs are dealt with under the civil procedure rule

Costs are discretionary on the courtDraws a distinction between civil procedure rules and other legislation

Costs are incidental to the rights of the party and are a defining part of the civil litigation process thus are seen as being procedural

Relate to the machinery and remedy as opposed to the right itselfEven though it affects parties, more focused on operating the system

Page 89: INTRODUCTION - University of Victoria - Web.UVic.caweb.uvic.ca/~lssweb/wp-content/outline_uploads/76... · Web viewDow Corning (2000) (BCCA) Facts: Class actions for negligence in

(efficiency, incentives) and it is discretionary

Pre and post judgment interest: substanceInterest arises from the Court of Justice Act, rather than being entirely at the discretion of the court (like costs)

While there is some discretion regarding prejudgment interest, it is much more fettered than with costs, which is entirely at the discretion of the courts

More importantly, prejudgment interest arises from statute (unlike costs which arise from the rules of civil procedure) = substantive

Presumptive right to this, even though it is not absolutePrimarily compensatory tool as opposed to costs which are about operating the system

Cap on non-pecuniary loss: proceduralGeneral conflict of laws principles regarding damages distinguish between an entitlement to damages and the quantification or measurement of damages:

Remoteness and heads of damage are questions of substance governed by the lex loci delicti, Whereas the quantification or measurement of damages is a question of procedure governed by the lex fori.

The cap was not a bar to the non-pecuniary damages, but was only a measure of the loss recovery. Thus, cap goes to the quantum and is therefore a procedural matter

Issue 2: International exception to lex loci delecti (more important)

Two arguments:Application of the foreign law would result in an injustice (but injustice must be quite significant and go beyond a mere difference in law)There is an insufficient connection to the lex loci delecti to apply the foreign law

OCA looks at Wong v. Lee (OCA): Facts: Single car accident in New York to a car registered in Ontario. All of the parties were from Ontario. Insurers were all Ontarian. Reasons:

"It is not mere differences in public policy that can ground the exception to the general rule of lex loci delicti; the exception is only available in circumstances where the application of the general rule would give rise to an injustice. Every difference in the laws of the two forums is going to benefit one side or the other and be perceived as unjust to the one not benefiting. Because La Forest J. [in Tolofson] anticipated the exercise of discretion being necessary only in a very unusual case, an injustice that would require a court to exercise the discretion must be something beyond ordinary differences between the laws of the forums.

La Forest J. did not articulate the criteria he envisaged for any particular circumstance to qualify as an injustice.

Held: Lex loci applies notwithstanding the high degree of connection between the parties and the forum

Page 90: INTRODUCTION - University of Victoria - Web.UVic.caweb.uvic.ca/~lssweb/wp-content/outline_uploads/76... · Web viewDow Corning (2000) (BCCA) Facts: Class actions for negligence in

All of the parties were residents of the forum and had no connection with the foreign jurisdiction. However, even in these circumstances the lex loci delecti governed.

In Somers, the court adopts the narrow approach to injustice from Wong (i.e. narrow international exception from Tolofson)

There were significant connections with NY: NY driver, insurance company, location of the accidentFurther, there was no substantive injustice to Somers with applying lex loci delecti - no real substantial injustice in part because there is a significant connection to NY

Comment: For injustice to occur, a whole avenue of damages would likely have to be cut off by the application of lex loci delecti

Ted thinks that the dissent in Wong will likely be adopted in the future (but does not govern now)Wong dissent: Have to interpret Tolofson cautiously – lex loci delecti should not be arbitrarily applied. After all, the context of Tolofson was interprovincial!

On the level of comity, it is difficult to see what interest New York State would have in applying its statutory compensation regime to a claim between two non-resident parties litigating in their home forum. In my opinion, the better view is that Ontario has a greater interest in the compensation of an Ontario resident by another Ontario resident being determined by Ontario law, rather than by New York law.An Ontario resident injured in a foreign state by the negligence of another Ontario resident, each of whom was insured in Ontario by an Ontario insurer, should be compensated under the Ontario statutory no-fault compensation regime for personal injuries sustained in a motor vehicle accident. To insist on the application of the lex loci delicti in this fact paradigm would preclude an Ontario court from awarding damages in compliance with Ontario's statutory compensation regime and would, thus, result in giving rise to an injustice of the nature contemplated in Tolofson.

Really a question of which state has the greater interest in the outcome of the case!

Enforcement of JudgementsStarting point for recognition and enforcement of foreign (non-BC) judgements surrounds the concept of territorial sovereignty.

Exclusive territorial sovereignty is what prevents the judgement from one jurisdiction from being enforced in another. Recognition and enforcement of a foreign judgement is dependant upon the laws and rules of the foreign state being enforced in BC forum court has to make a judgement on its own about whether the foreign judgement should be enforced.

Page 91: INTRODUCTION - University of Victoria - Web.UVic.caweb.uvic.ca/~lssweb/wp-content/outline_uploads/76... · Web viewDow Corning (2000) (BCCA) Facts: Class actions for negligence in

While there are limitations on the enforcement of foreign judgements, there is a clear need for the regular recognition and enforcement of foreign judgements:

Without this, every transnational dispute would have to be relitigated multiple times. Anytime there was contract with multiple foreign parties, would have to litigate in each jurisdiction! There are thus strong policy reasons for the recognition and enforcement of foreign judgements.

Two types of enforcement of judgements: in rem and in personamIn personam judgements will be our focusThere are different rules for in rem judgements (won’t be covered)

Enforcement of Foreign JudgementsIn BC and across the provinces, foreign judgements can be recognized and enforced in two ways:

Bring an action in a BC court on the foreign judgement as a debt Essentially enforcing the foreign judgement that created the debtCommon law approach treats in personam claims as enforcement of debt

Register and enforce a judgement through legislationIn BC, you have two choices: Court Order Enforcement Act (older – likely falling into obscurity) Enforcement of Canadian Judgements and Decrees Act –

restricted to Canadian judgements (new)

For Canadian judgements, more likely to go under the legislations – tends to streamline the process and eliminate certain stepsFor international judgements, more likely to use the common law – legislation does not always apply to non-Canadian judgements

NOTE: Statute and common law can work together simultaneouslyStatute does not override the common law – you can choose to use the common law over the statute!

Limitation periods: NEW RULE in BC for the enforcement and recognition of non-BC judgements: limitation period to be applied is the one from where the judgement originally arose, or 10 years. – whether you are going under the common law or the statute

What if you can’t get the judgement enforced?You can relitigate the substantive matter in the local forum (as long as you won in the foreign forum) – but have to go through jurisdiction simpliciter, FNC, etc.But if you lost in another forum, you can’t just relitigate the whole thing – abuse of process

Page 92: INTRODUCTION - University of Victoria - Web.UVic.caweb.uvic.ca/~lssweb/wp-content/outline_uploads/76... · Web viewDow Corning (2000) (BCCA) Facts: Class actions for negligence in

Common Law EnforcementThere are two requirements for a foreign judgement to be recognized and enforced under common law:

Judgement from the foreign court must be final and conclusiveForeign court making the judgement had to have jurisdiction “in the international sense” to decide the matter. Three ways to meet this requirement:

Presence: If the defendant was present in the foreign jurisdiction when the action commenced Attornment: If the defendant attorned to the jurisdiction of the foreign courtCanadian addition - Morguard – if in the foreign court there is a real and substantial connection between the facts and the exercise of its jurisdiction

Final and ConclusiveNouvion v. Freemen (1899) (HoL)

Facts: Suit in England to enforce a court order from Spain that debt is owing.Issue: whether the Spanish judgement was “final and conclusive”?Test: can the court order be changed by the same court that announced it?

Inquiry is focused on the court that made the decision - court distinguished between courts still seized with the matter being different from where there is an appeal of the case going on.If there was a process that allowed the person to come back to re-argue the case, it is not final and conclusiveJudgment must meet the res judicata principle: there has been a final judgment that is no longer subject to appealA right of appeal or an appeal proceeding does not prevent the judgment from being final and conclusive

Held: enforcement of Spanish judgment denied the same matter could be litigated by the same parties before same court not final and conclusive

NOW: Appeals are dealt with under the BC legislation and the Rules of Court :

If someone brings a foreign judgement they want enforced in BC and there is an appeal in process, then the opposing party can ask for a stay until the appeal has been dealt with in the foreign jurisdiction. But to get a stay, the appeal has to have been commenced or there must be a clear intention to appeal.

Jurisdiction “in the international sense”Jurisdiction in the international sense is the bigger issue:

Courts generally reluctant to enforce a judgement from a strange place – don’t want to recognize if the foreign court didn’t have suitable jurisdiction But how far behind the judgement do you look?

If it’s a Canadian judgement, don’t need to look to deep because you can assume that the decision is valid

Page 93: INTRODUCTION - University of Victoria - Web.UVic.caweb.uvic.ca/~lssweb/wp-content/outline_uploads/76... · Web viewDow Corning (2000) (BCCA) Facts: Class actions for negligence in

Jurisdiction in the international sense is not the same thing as jurisdiction simpliciter, but it often looks like the same thing

Rather a matter of determining whether the foreign judgement is one that should be recognized in BC

Morguard (2002) (SCC)Facts: Plaintiff was the mortgagee of lands in Alberta. Morgatgor moved to BC and went into default. Suit brought in Alberta to recover the money. BC defendant took no action. Mortgage was foreclosed and everything was sold off. But the BC defendant still owed more money.

Defendant was not in Alberta when the action commencedDefendant did not attorne to the Alberta court jurisdictionRelevant BC legislation also did not allow for enforcement

BC trial court enforced the Alberta judgement (probably wrong)BCCA dismissed the appeal SCC disagrees with the CA on reasoning: kick out the old historic British rules, or at least add to them:

Majority (La Forest): One of the basic tenets of international law is that sovereign states have exclusive jurisdiction over their own territory and are therefore reluctant to enforce judgements from foreign jurisdiction.

Examines the historical English rules: presence and submission (attornment)Criticizes 100 year of Canadian jurisprudence – applied English law blindly without adapting it to Canadian needs: the lack of comity in the old English cases reflected Old World order and didn’t allow for the fact that times are changing

Why is it necessary to change the law?It is anarchic and unfair that a person should be able to avoid legal obligations arising in one province simply by moving to another province.

Why should a plaintiff be compelled to begin an action in the province where the defendant now resides, whatever the inconvenience and costs this may bring, and whatever degree of connection the relevant transaction may have with another province? And why should the availability of local enforcement be the decisive element in the plaintiff's choice of forum?

Recognition in other provinces should be dependent on the fact that the court giving judgment "properly" or "appropriately" exercised jurisdiction.

Held: “More real and substantial connection can scarcely be imagined” - Alberta court had jurisdiction and should be recognizable and enforceable in BC.

In this case, difficult to imagine a more reasonable place for the action to take place than Alberta:

Contract in AlbertaProperty in Alberta

Page 94: INTRODUCTION - University of Victoria - Web.UVic.caweb.uvic.ca/~lssweb/wp-content/outline_uploads/76... · Web viewDow Corning (2000) (BCCA) Facts: Class actions for negligence in

At the time, all parties resident in Alberta

Consideration: inconvenience to the defendant to have to attend action in Alberta

“It seems to me that the approach of permitting suit where there is a real and substantial connection with the action provides a reasonable balance between the rights of the parties.

It affords some protection against being pursued in jurisdictions having little or no connection with the transaction or the parties. In a world where even the most familiar things we buy and sell originate or are manufactured elsewhere, and where people are constantly moving from province to province, it is simply anachronistic to uphold a "power theory" or a single situs for torts or contracts for the proper exercise of jurisdiction.”

Result: New criteria for recognition and enforcement of judgements: real and substantial connection

Certainly applies within Canada based on full faith and credit analysis and constitutional analysisBut does it apply to international cases?

Braintech v. Kostiuk (1999) (BCCA)Facts: Plaintiff obtained a default judgement against the defendant, a BC resident, for libel and defamation. Defendant had allegedly placed material on a group bulletin board on the Internet that was defamatory or libellous about the plaintiff. Plaintiff had some kind of presence in Texas, sufficient to bring an action in a Texas court against the defendant (Statute directed that even though the defendant was not physically present in Texas, because they were carrying on business there, this was sufficient presence.). Texas court based its jurisdiction its own civil procedure rules which provided that Texas court had jurisdiction simpliciter regarding non-residents doing business in Texas and where the non-resident commits a tort in Texas. Plaintiff, having gotten the judgement, comes to BC to enforce it against the BC resident.

Plaintiff was not phsycially present in Texas and did not attorne to their jurisdiction

Held: There had been no presence and no submission/attornement.

Presence: BCCA rejected the Texas court’s determination that the defendant was “present” in Texas

Lex fori gets to determine the question of presence BC court doesn’t have to accept the Texas determinationHere, says that the defendant was NOT present in Texas

Attornment: defendant had clearly not attorned to the jurisdiction of Texas, even if they had been served.

Thus, apply the Morguard principles:In Texas, court had determined that there was a real and substantial connection because affected Texas resident and “took place in Texas” (but we’re on the Internet!)

Page 95: INTRODUCTION - University of Victoria - Web.UVic.caweb.uvic.ca/~lssweb/wp-content/outline_uploads/76... · Web viewDow Corning (2000) (BCCA) Facts: Class actions for negligence in

Court references the American case of Zippo Manufacturing v. Zippo Dotcom (1997): “…the likelihood that personal jurisdiction can be constitutionally exercised is directly proportionate to the nature and quality of commercial activity that an entity conducts over the Internet.

At one end of the spectrum are situations where a defendant clearly does business over the Internet. If the defendant enters into contracts with residents of a foreign jurisdiction that involve the knowing and repeated transmission of computer files over the Internet, personal jurisdiction is proper. ... At the opposite end are situations where a defendant has simply posted information on an Internet Web site which is accessible to users in foreign jurisdictions. A passive Web site that does little more than make information available to those who are interested in it is not grounds for the exercise personal jurisdiction.

BCCA adopts this approach: BC resident fit into the passive category – simply posting information

“To enforce recovery of the default judgment obtained in Texas on the deemed proof of use of an electronic bulletin board would encourage a multiplicity of actions the world over wherever Internet was available.”

Held: British Columbia was the only natural forum, and Texas was not an appropriate forum. Comity did not require the courts of British Columbia to recognize the default judgment.

Will not enforce the judgement because Texas did not take jurisdiction properly – goes so far as to say that Texas court asserted jurisdiction unconstitutionally under US law!

Beals v. Saldanha (2003) (SCC)Facts: Default judgement from a Florida court regarding a minor or honest error on a land purchase. Ontario family had bought land in Florida. There was a problem with the lot that had been purchased. They were going to sell the land, and sold the wrong lot by accident (sort of). The actual sales agreement got fixed. Purchaser was a real estate agent perhaps tried to take advantage of the mistake. In Florida, was awarded $1 million in damages.

No presence or submission (attornment)Florida court had jurisdiction in the international sense – not at issue in this case. Issue rather surrounded the defences.

All of the judgements reaffirm that Morguard does apply to foreign judgements!

Real and substantial connection: conceded that Florida had jurisdiction, so the case was argued on the exceptions: fraud and natural justice

Majority (Majer): Morguard should apply to international judgementsFlow of goods and people necessitates liberal enforcementReciprocity: if the same facts occurred, would Ontario have taken jurisdiction or not? Yes – therefore, not unreasonable that Florida took jurisdiction

Not whether Florida would recognize the judgement , but whether Ontario would have taken jurisdiction

Page 96: INTRODUCTION - University of Victoria - Web.UVic.caweb.uvic.ca/~lssweb/wp-content/outline_uploads/76... · Web viewDow Corning (2000) (BCCA) Facts: Class actions for negligence in

Not abnormal part of the conflicts of law analysis

Ontario family purchased land in Florida = significant engagement with the foreign jurisdiction

“Where a party takes such positive and important steps that bring him or her within the proper jurisdiction of a foreign court, the fear of unfairness related to the duty to defend oneself is lessened. If a Canadian enters into a contract to buy land in another country, it is not unreasonable to expect the individual to enter a defence when sued in that jurisdiction with respect to the transaction.

Concurring (Binnie): Agrees with the majority that Morguard should apply internationally, but thinks it should NOT be applied blindly to international judgements:

There is something different about judgements from another Canadian jurisdiction and from an international jurisdiction (internal vs. external judgements).

Dissent (Lebel): Again, agrees that Morguard should be applied internationally, but that we must distinguish between Canadian judgement and non-Canadian judgements

Although I agree both that the "real and substantial connection" test should be extended to judgments from outside Canada and that the Florida court properly took jurisdiction over the defendants in this particular case, in my view the test should be modified significantly when it is applied to judgments originating outside the Canadian federation. Specifically, the assessment of the propriety of the foreign court's jurisdiction should be carried out in a way that acknowledges the additional hardship imposed on a defendant who is required to litigate in a foreign country.

“Context sensitive jurisdiction test”: must take into account the difficulty of defending in a foreign jurisdiction. Consider:

Quality of the justice in the foreign systemReal and substantial connection in the international sense impacted by convenience to the defendantHow real and substantial a does a connection have to be?

Must be strong enough to make it reasonable for the defendant to have expected to have litigated there, even though it may entail additional expense, risk, time, etc.

If litigating very burdensome, a stronger degree of connection is requiredList of burdens to be taken into account:

ExpensesInconvenience of travelFinding out the foreign lawCorruption and bias

But in this case, there was a very strong connection with Florida: any potential burden is outweighed by the strength of the connection!

Page 97: INTRODUCTION - University of Victoria - Web.UVic.caweb.uvic.ca/~lssweb/wp-content/outline_uploads/76... · Web viewDow Corning (2000) (BCCA) Facts: Class actions for negligence in

Reciprocity argument: rejects based on an equivalence of jurisdiction

Difficulty of finding a balance between enforcing foreign judgements and protecting Canadian citizens:

The traditional rules impeded cross-border commerce by making it difficult for judgment creditors to obtain effective remedies against defendants resident in other countries, thus undermining the security of transactions. But an excessively generous test would be unduly burdensome for defendants and might discourage persons with assets in Canada from entering into transactions that could eventually get them involved in international disputes. Ideally, the test should represent a balance designed to create the optimum conditions favouring the flow of commodities and services across state lines. Judgements should travel more easily across provincial boundaries than national boundaries

Don’t blindly apply Morguard to foreign judgements:“The "real and substantial connection" test should apply to foreign-country judgments, but the connections required before such judgments will be enforced should be specified more strictly and in a manner that gives due weight to the protection of Canadian defendants without disregarding the legitimate interests of foreign claimants.”

Comment: Ted thinks that with the right set of facts, Lebel dissent likely to get picked up by the majority.

Defences in BealsFocus of Beals is really on the defences:

Public policy defence (already dealt with)FraudLack of natural justice

All of these defences arose at a time when getting enforcement of foreign judgements was moderately difficult before Morguard.

Given that Morguard is a much broader test that allows for the greater enforcement of foreign judgement, we don’t want to be enforcing too many judgements - may result in unfairness. Might be a good idea to:

Revisit the defences, OROld defences very narrow – perhaps need to be opened up

Think about a new type of defence New defences for the new context of Morguard

Fraud: has the court taken jurisdiction fraudulently?The merits of a foreign judgment can be challenged for fraud only where the allegations are new and not the subject of prior adjudication.

Must be new information that has been made available that clearly indicates that fraud has taken place

Held: not sufficient evidence of fraud (although there probably was!)Lebel: much more in favour of opening up the defences, but fraud here wasn’t made out

Page 98: INTRODUCTION - University of Victoria - Web.UVic.caweb.uvic.ca/~lssweb/wp-content/outline_uploads/76... · Web viewDow Corning (2000) (BCCA) Facts: Class actions for negligence in

Natural justiceMinimum standard of fairness of process: given adequate notice of the proceedings and given an opportunity to defend yourself

Difficult here because the Ontario family chose not to defend in the Florida case

BUT Ontario family had insufficient notice of what their true financial liability would be argued that although they had notice of the proceedings, they had not idea that it could end up being so expensive!Held: really no issue of the insufficiency of the proceedings to amount to a breach of natural justice.

This is where Lebel disagrees on the result:Notice requires information of the monetary liabilityIndicates that this is a problem in this case the family really didn’t appreciate the depth of their potential liability

Essentially argued that this would shock the conscience of the public: family didn’t have this much money, no insurance, etc.

Would not enforce the judgment on these grounds

Read Old North State vs. Newlands